52
©2015 Pieper Bar Review 1 CONTRACTS MNEMONICS 1) The ingredients for a valid contract are TACO: T Definite TERMS, express or implied A ACCEPTANCE of terms C CONSIDERATION O OFFER inviting acceptance 2) An offer expires when it gets TIRED: T Reasonable TIME after an offer is made, or after expiration date expressly stated in an offer I Mental INCAPACITY or death of offeror or offeree R REVOCATION of an offer communicated to an offeree before acceptance E EXPRESS or implied rejection communicated to offeror D DESTRUCTION of the subject matter of the offer or intervening illegality terminates an offer by operation of law 3) Options can DIE by: D DESTRUCTION of subject matter I Intervening ILLEGALITY E EXPIRATION of a stated option time extinguishes the option 4) In NY, a signed writing takes the place of consideration for POP: P PREEXISTING duty O Contract OPTIONS P PAST consideration (which must be recited in the signed writing) 5) Generally, contracting parties are free to modify a 3 rd party beneficiary (3PB) K, unless, prior to receiving notice of the K modification, the 3PB got MAD: M MANIFESTED an assent called for in the 3PB K, at the request of one of the contracting parties (i.e., accepted a K offer arising from the 3PB K) A Commenced a breach of K ACTION against the promisor, or D DETRIMENTALLY relied on the K

CONTRACTS MNEMONICS - Pieper Bar Review › ... › CWS15-Contracts-Mnemonics.pdfcontracting parties (i.e., accepted a K offer arising from the 3PB K) A – Commenced a breach of K

  • Upload
    others

  • View
    6

  • Download
    1

Embed Size (px)

Citation preview

Page 1: CONTRACTS MNEMONICS - Pieper Bar Review › ... › CWS15-Contracts-Mnemonics.pdfcontracting parties (i.e., accepted a K offer arising from the 3PB K) A – Commenced a breach of K

©2015 Pieper Bar Review 1

CONTRACTS MNEMONICS

1) The ingredients for a valid contract are TACO:

T – Definite TERMS, express or implied

A – ACCEPTANCE of terms

C – CONSIDERATION

O – OFFER inviting acceptance

2) An offer expires when it gets TIRED:

T – Reasonable TIME after an offer is made, or after expiration date expressly

stated in an offer

I – Mental INCAPACITY or death of offeror or offeree

R – REVOCATION of an offer communicated to an offeree before acceptance

E – EXPRESS or implied rejection communicated to offeror

D – DESTRUCTION of the subject matter of the offer or intervening illegality

terminates an offer by operation of law

3) Options can DIE by:

D – DESTRUCTION of subject matter

I – Intervening ILLEGALITY

E – EXPIRATION of a stated option time extinguishes the option

4) In NY, a signed writing takes the place of consideration for POP:

P – PRE–EXISTING duty

O – Contract OPTIONS

P – PAST consideration (which must be recited in the signed writing)

5) Generally, contracting parties are free to modify a 3rd

party beneficiary (3PB) K, unless, prior to

receiving notice of the K modification, the 3PB got MAD:

M – MANIFESTED an assent called for in the 3PB K, at the request of one of the

contracting parties (i.e., accepted a K offer arising from the 3PB K)

A – Commenced a breach of K ACTION against the promisor, or

D – DETRIMENTALLY relied on the K

Page 2: CONTRACTS MNEMONICS - Pieper Bar Review › ... › CWS15-Contracts-Mnemonics.pdfcontracting parties (i.e., accepted a K offer arising from the 3PB K) A – Commenced a breach of K

©2015 Pieper Bar Review 2

6) Contract assignments may involve the ADA:

A – ASSIGNMENT of a contractual right to collect money owed under the K

D – DELEGATION of the performance required under the K

A – ASSUMPTION of liability for performing the K

7) A gratuitous assignment becomes irrevocable, and a second assignee prevails over a prior assignee of

the contract when J.P.N.C.:

J – Recovers a JUDGMENT

P – Gets PAID

NC – Enters a NEW CONTRACT

8) Absent express language in a K prohibiting assignment, K rights are freely assignable, except those of

SIR P:

S – Where a STATUTE expressly prohibits the assignment of a K right (but if that

claim is reduced to judgment, it is assignable)

I – Where the assignment is coupled with an IMPROPER delegation of a duty

under the K to a person unqualified to fulfill that duty

R – Where the assignment increased the RISK to the other contracting party

P – Where the services to be rendered are highly PERSONAL in nature (because

that would materially alter the bargain)

9) In New York, by statute (see SIR P), you cannot assign a WASP:

W – WORKER’S COMPENSATION

A – ALIMONY or child support payments

S – SPENDTHRIFT TRUSTS

P – PERSONAL INJURY or wrongful death causes of action

10) Look at HAIL to determine whether a breach is material or immaterial:

H – HARDSHIP on breaching party if total material breach is declared

A – AMOUNT of benefit bestowed on non–breaching party

I – Whether breach was INNOCENT

L – LIKELIHOOD of full performance being achieved

11) Breach of contract defenses are I3

FU2MED & I S

2IP:

I – INFANCY

I – INSANITY – INCOMPETENCY

I – INTOXICATION

F – FRAUD

U – UNCONSCIONABILITY

U – UNDUE INFLUENCE

M – MISTAKE

E – EQUITABLE DEFENSES

D – DURESS

I – IMPOSSIBILITY of performance

S – STATUTE OF FRAUDS

S – STATUTE OF LIMITATIONS

I – ILLEGALITY

P – PAROLE EVIDENCE RULE

Page 3: CONTRACTS MNEMONICS - Pieper Bar Review › ... › CWS15-Contracts-Mnemonics.pdfcontracting parties (i.e., accepted a K offer arising from the 3PB K) A – Commenced a breach of K

©2015 Pieper Bar Review 3

12) Lack of contractual capacity arises from the 3 I’s:

I – INFANCY

I – INTOXICATION

I – Mental INFIRMITY

13) SI2R M is a fraud:

S – SCIENTER

I – D lied with an INTENT to defraud the P

I – P suffered an economic INJURY

R – P justifiably RELIED on D’s misrepresentation

M – D misrepresented a MATERIAL fact, which induced P to enter the K

14) A unilateral mistake in calculating figures may allow the mistaken party the remedy of equity of

rescission, if he calls the COPS:

C – The computational mistake was COMMUNICATED to the other party before

that person changed his/her position in reliance on those mistaken figures

O – The mistake involved was one of ORDINARY negligence

P – The mistaken party gave PROMPT notice of the mistake

S – The mistake will impose SUBSTANTIAL hardship on the party if not

corrected

15) The following SMART FLYS contracts must be in writing, subscribed by the party to be charged

with the breach (i.e., must contain defendant’s signature):

S – SURETY contracts

M – MARRIAGE contracts

A – ANSWER for debts discharged in bankruptcy

R – REAL ESTATE contracts

T – TESTAMENTARY promises (NY ONLY)

F – FINDERS FEE arrangements

L – LEASES longer than 1 year

Y – Contracts not capable of complete performance within 1 YEAR

S – UCC Article 2 SALES CONTRACTS for $500 or more

16) Use a COMB for promissory estoppel in NY:

C – CHARITABLE pledges

O – To avoid OUTRAGEOUSLY unconscionable results

M – Oral MARRIAGE contracts

B – Promises by gratuitous BAILEES to obtain insurance on bailed goods

17) There are 4 T–CUP elements for a constructive trust:

T – TRANSFER of property in reliance on promise

C – Existence of CONFIDENTIAL or fiduciary relationship

U – UNJUST enrichment to transferee of property or to some third party, AND

P – PROMISE, express or implied, to hold property for plaintiff’s benefit, which

promise has been breached

Page 4: CONTRACTS MNEMONICS - Pieper Bar Review › ... › CWS15-Contracts-Mnemonics.pdfcontracting parties (i.e., accepted a K offer arising from the 3PB K) A – Commenced a breach of K

©2015 Pieper Bar Review 4

18) A THUG may render an illegal contract enforceable, based on:

T – TYPE of illegality & extent to which the public is harmed

H – HARM that forfeiture would cause if contract was declared unenforceable due

to illegality; ct looks to see whether contract has been substantially performed

U – UNJUST enrichment (a windfall) to party asserting illegality defense

G – Relative GUILT of each party

19) The theory of impossibility frequently involves the 4 Ds:

D – DEATH

D – DANGER to life/ill health

D – DESTRUCTION of the subject matter of the law suit

D – DELAYS, temporarily causing performance to become impracticable or

impossible

20) OF MICE2 permits parole evidence:

O – To establish an ORAL condition precedent to legal effectiveness of contract,

provided it doesn’t contradict express term(s) of the contract

F – A party cannot invoke the Parole Evidence Rule to shield that party from

allegations of FRAUD or Misrepresentation

M – To establish MUTUAL Mistake or claim for reformation of contract

I – To establish ILLEGALITY

C – To establish failure of CONSIDERATION

E – To EXPLAIN ambiguous or missing terms

E – To show that no ENFORCEABLE agreement was ever intended

21) Contract law does not allow damages recovery for CAPS:

C – To recover consequential damages, unless they were within the

CONTEMPLATION OF BOTH PARTIES when the contract was executed

A – Damages that party could have AVOIDED

P – Damages for PAIN & suffering or emotional distress resulting from a

breached contract, even if such damages were foreseeable

S – SPECULATIVE damages aren’t recoverable (all damages must be proven

within a reasonable certainty)

22) Generally, parties can put whatever terms they’d like into a K, except for PLUS:

P – Terms that violate PUBLIC POLICY

L – Terms providing for an excessive amount of LIQUIDATED DAMAGES

U – Terms that are UNCONSCIONABLE

S – Clauses providing that one party can seek SPECIFIC PERFORMANCE in

the event of a breach (the contract does NOT have to enforce these clauses)

23) Apply a TISSUE to a covenant restricting a former employee from competing:

T – TIME restriction must be reasonable (usually two years or fewer)

I – INABILITY of the employee to gain work elsewhere

S – The geographic SPACE/SCOPE of the restriction must be as narrow as

possible (must only be to the extent necessary to protect employer’s interest)

SUE – The employee services must be SPECIAL, UNIQUE, or

EXTRAORDINARY

Page 5: CONTRACTS MNEMONICS - Pieper Bar Review › ... › CWS15-Contracts-Mnemonics.pdfcontracting parties (i.e., accepted a K offer arising from the 3PB K) A – Commenced a breach of K

©2015 Pieper Bar Review 5

SALES MNEMONICS

24) ICOP limits the Perfect Tender Rule:

I – INSTALLMENT contracts

C – Timely delivery was COMMERCIALLY IMPRACTIBLE by an event not

contemplated by the parties.

O – Delivery in good faith, OBJECTIVELY and reasonably believing the goods

would be acceptable to the buyer

P – PRIOR TO DELIVERY DATE set forth in the contract, conforming goods

are delivered to replace the nonconforming goods

25) Additional terms will not be added to the contract when OCAN:

O – The offeror OBJECTS to additional terms within a reasonable time

C – The offer expressly CONDITIONS the agreement on accepting the terms in

the offer as they are

A – The additional terms materially ALTER the offer

N – Either or both parties are NON–MERCHANTS

26) A J STRAW clause materially alters an offer if it would cause surprise or hardship

to the offeror if the offeror was not made aware of its existence:

J – Bestowing JURISDICTION on a particular court, or requiring offeror to

consent to jurisdiction in particular state

S – Shortening the STATUTE OF LIMITATIONS to sue for non–conforming

goods

T – Limiting TORT liability or limiting a buyer’s right to sue for consequential

damages

R – Altering UCC rules for RISK OF LOSS

A – Adding an ARBITRATION CLAUSE (unless customary to do so in the

trade)

W – Adding a clause negating a WARRANTY (e.g., one of merchantability or

fitness)

27) Exceptions to the Statute of Frauds requirement are SWAMP:

S – Contracts for SPECIALLY manufactured goods

W – WAIVER A – Judicial ADMISSION of contract

M – “MERCHANT MEMORANDUM”

P – PART PERFORMANCE

28) If a sales contract is silent on a topic, the UCC implies the following CIDER rules:

C – Seller is not obligated to extend CREDIT to the buyer

I – Buyer has the right to INSPECT the seller’s tendered goods (except no right

to inspect when the transaction involves a bill of lading)

D – Seller’s tender of DELIVERY is implied to be at seller’s place of business,

unless both parties know that the goods are located elsewhere

E – Buyer and seller must EXCHANGE performance concurrently

R – RISK OF LOSS is on the party in the best position to bear that risk

Page 6: CONTRACTS MNEMONICS - Pieper Bar Review › ... › CWS15-Contracts-Mnemonics.pdfcontracting parties (i.e., accepted a K offer arising from the 3PB K) A – Commenced a breach of K

©2015 Pieper Bar Review 6

29) SOAL–V and SORE–V affect risk of loss:

SOAL V – SALE ON APPROVAL LATE VESTING (goods held by the

buyer are not subject to claims of the buyer’s creditors)

SORE V – SALE OR RETURN EARLY VESTING (title and ROL vest

immediately in the buyer, even though the buyer has a right to

rescind the K)

30) Remedies available to a seller are SPARKLE:

S – STOPPING goods in transit

P – Suing for the entire contract PRICE

A – Demanding ASSURANCES

R – RE–SELLING goods to another buyer

K – KEEPING part of a breaching buyer’s deposit, never more than $500

L – Suing for LOST Profit

E – EXERCISING the right to reclaim goods delivered to the insolvent buyer

31) Remedies available to a buyer are CID’S WAR:

C – COVER

I – INCIDENTAL & consequential damages

D – DAMAGES for lost benefit of the bargain, or for the price paid

S – SPECIFIC PERFORMANCE on a contract for unique goods

W – Breach of WARRANTY

A – ACCEPTANCE revocation

R – REJECTING non–conforming goods

32) A sales contract contains M FEET warranties:

M – Warranty of MERCHANTABILITY

F – Warranty of FITNESS for a particular purpose

E – Warranty against ENCUMBRANCES

E – EXPRESS warranties

T – Warranty of TITLE

33) Express warranties are SAD:

S – SAMPLE or model, which is the basis of the bargain

A – Written or oral AFFIRMATION of fact or promise made by the seller relating

to the goods

D – DESCRIPTION of the goods in advertisements, brochures, or catalogs

34) A P’s claim against a seller for a defective good can be based on one or more overlapping but

different PINE theories of liability:

P – Torts theory of strict PRODUCTS liability

I – Contract theory for breach of IMPLIED warranty

N – Torts theory of NEGLIGENCE

E – Contract theory for breach of an EXPRESS warranty

Page 7: CONTRACTS MNEMONICS - Pieper Bar Review › ... › CWS15-Contracts-Mnemonics.pdfcontracting parties (i.e., accepted a K offer arising from the 3PB K) A – Commenced a breach of K

©2015 Pieper Bar Review 7

35) G.P.S. LAMP can use the following defenses against a breach of warranty claim:

G – GOVERNMENT military contract defense

P – Federal PREEMPTION

S – STATUTE OF LIMITATIONS

L – LACK of timely notice to a seller

A – ASSUMPTION of risk (can be asserted against any PINE claim)

M – Unforeseen MISUSE of a product

P – Lack of PRIVITY of contract

36) Strict products liability is imposed on a regular seller of a DUD product:

D – DEFECTIVE, and an

U – UNREASONABLY D – DANGEROUS product

37) When asserting a strict products liability claim, P must prove that a DIM dangerous defect in the

product proximately caused a physical injury:

D – DESIGN defect

I – INADEQUATE warning

M – Mistake in the MANUFACTURING process

Page 8: CONTRACTS MNEMONICS - Pieper Bar Review › ... › CWS15-Contracts-Mnemonics.pdfcontracting parties (i.e., accepted a K offer arising from the 3PB K) A – Commenced a breach of K

©2015 Pieper Bar Review 8

Damages The Goal: Contract law seeks to provide the non–breaching party with its Expectation Damages,

which place the non–breaching party in as good a position as if the breaching party had performed its

obligations under the contract. Expectation Damages are the usual form of damages awarded when a

contract is breached.

The General Formula:

The Value of the Promised Performance

MINUS The Value of what the Plaintiff Received

PLUS

Any Incidental and/or Consequential Damages

MINUS

Any Costs that Were/Could have been

Avoided/Mitigated

To make use of this formula, it helps to know something about each of its component parts:

The “value of the promised performance” is typically just the contract price, though a fact pattern could

tell you that the value of the performance was something else, for example, if the buyer was getting a

good deal/discounted price. By allowing a plaintiff to recover the value of the promised performance as

opposed to just the contract price, the plaintiff’s “benefit of the bargain” is preserved.

The “value of what the plaintiff received” is exactly what it says: some value associated with the

defendant’s performance (this value will be identified in the fact pattern, if necessary).

“Incidental damages” are any “costs incurred in a reasonable effort, whether successful or not, to avoid

loss, as where a party pays brokerage fees in arranging or attempting to arrange a substitute transaction.”

Restatement 2d Contracts § 347, comment c. In a sale of goods contract, incidental damages are awarded

to a non–breaching buyer for any reasonable expenses arising out of the breach including those incurred

through cover, or any cost “reasonably incurred in inspection, receipt, transportation and care and custody

of goods rightfully rejected . . . .” UCC 2–715. A non–breaching seller can recover incidental damages

for charges incurred in “stopping delivery, in the transportation, care and custody of goods after the

buyer's breach, [or] in connection with return or resale of the goods” etc. UCC 2–710.

“Consequential damages” are additional losses incurred by the plaintiff as a result of the defendant’s

breach, that usually arise in the area of lost profits following the defendant’s failure to perform on time.

To recover consequential damages, the plaintiff must show 1) causation (the damages were a result of the

defendant’s breach), 2) the damages were foreseeable at the time the parties entered into the contract, 3) a

reasonable certainty as to the amount of damages, and 4) that the damages could not have been mitigated.

Page 9: CONTRACTS MNEMONICS - Pieper Bar Review › ... › CWS15-Contracts-Mnemonics.pdfcontracting parties (i.e., accepted a K offer arising from the 3PB K) A – Commenced a breach of K

©2015 Pieper Bar Review 9

“Costs that were or could have been avoided or mitigated” are simply 1) costs that the plaintiff will no

longer have to incur following the defendant’s breach (sometimes referred to as “costs avoided”), like any

amounts that the plaintiff no longer has to pay under the breached contract, and 2) losses that the plaintiff

can mitigate “by making substitute arrangements for the use of his [or her] resources that are no longer

needed to perform the contract” (sometimes called “losses avoided”) Restatement 2d Contracts § 347,

comment d.

To avoid pitfalls, always remember that contract law prohibits the recovery of CAPS:

C – CONSEQUENTIAL DAMAGES, unless they were within the contemplation of both parties

(foreseeable) when the contract was executed

A – Damages that party could have AVOIDED

P – Damages for PAIN & suffering or emotional distress resulting from a breached contract, even if such

damages were foreseeable

S – SPECULATIVE damages (all damages must be proven within a reasonable certainty.

The Caveat: The formula and its component principles are a guide. Any damages formula will

work well in some situations, but not work well in others. Our job is to think about concepts like placing

the non–breaching party in as good a position as performance would have put her in, and not awarding

damages for costs that could have been avoided, and then apply them as rationally as possible. By

working through the scenarios to follow, you should gain an understanding of how the courts award

damages in a range of different situations, so that you’ll be able to answer any damages questions on the

bar exam.

Alternate Method of Looking at Damages: Scholars have developed an

alternate measure of damages (not yet embraced by the courts but tested on the Multistate Bar Exam, see

e.g. OPE 3 Q98) which breaks down the plaintiff’s recovery into expectation, restitution, and reliance

interests. See Joseph M. Perillo, Calamari and Perillo on Contracts 490 (6th

ed. 2009).

Expectation Interest = the plaintiff’s expected gain under the contract (essentially lost profits)

Restitution Interest = benefits conferred on the defendant that the plaintiff is entitled to recover

Reliance Interest = the economic detriment incurred by the plaintiff as a result of the breached contract,

which typically includes the restitution interest.

This alternate, modern method embraces the same goal of placing the non–breaching party in as good a

position as if the breaching party had performed its obligations under the contract, but uses these interests

to categorize the elements of the plaintiff’s recovery. These terms are based on the traditional measures

of damages (reliance and restitution).

Page 10: CONTRACTS MNEMONICS - Pieper Bar Review › ... › CWS15-Contracts-Mnemonics.pdfcontracting parties (i.e., accepted a K offer arising from the 3PB K) A – Commenced a breach of K

©2015 Pieper Bar Review 10

Examples:

PROBLEM #1: Breach by Paying Party

Construction Contract Where Homeowner Breaches

1(a). A builder agreed with a homeowner to build a house for $100,000, which would have generated a

$10,000 profit for the builder. If the homeowner repudiated the contract before the builder began, then

the builder could sue for his lost bargain of $10,000, which is the value of the performance (here, the

contract price of $100,000), minus what the builder received (here, nothing, since the fact pattern gives no

indication that the homeowner was paid anything), minus the cost of completion (here, the $90,000

expenditure avoided by the builder not having to complete performance).

Under the alternate, modern method, the damages would be the same, but the $10,000 would categorized

as the plaintiff’s expectancy interest.

1(b). If the builder had partially performed the contract, spending $60,000 on construction prior to the

homeowner’s repudiation (anticipatory breach), the builder’s damages would be $70,000, i.e., the value of

the performance (here, the contract price of $100,000), minus what the builder received (here, nothing,

since the fact pattern gives no indication that the homeowner was paid anything), minus the cost of

completion (here, the remaining $30,000 expenditure avoided by the builder not having to complete

performance).

Under the alternate, modern approach, the amount needed to make the builder whole would still be

$70,000, consisting of a $60,000 reliance interest, plus a $10,000 expectancy interest.

1(c). If the builder had completed the contract, spending $90,000, then he would be entitled to the entire

contract price. That could be measured by the value of the performance (here, the contract price of

$100,000), minus what the builder received (here, nothing, since the fact pattern gives no indication that

the homeowner was paid anything), minus the cost of completion (here, nothing, since the house was

completed).

Under the alternate, modern approach, the amount needed to make the builder whole would still be

$100,000, consisting of a $90,000 reliance interest, plus a $10,000 expectancy interest.

Page 11: CONTRACTS MNEMONICS - Pieper Bar Review › ... › CWS15-Contracts-Mnemonics.pdfcontracting parties (i.e., accepted a K offer arising from the 3PB K) A – Commenced a breach of K

©2015 Pieper Bar Review 11

PROBLEM #2: Construction Contract Where Builder Breaches

Owner Recovers the Cost of Completion

2(a). A builder agreed with a homeowner to build a basic frame house for $100,000. The homeowner

paid the builder upfront, $100,000. After doing most of the job, the builder repudiated (breached) the

contract. Other builders would have charged the homeowner $5,000 to finish the job, but the homeowner

found builder X, who needed work and was willing to do it for $4,000. How much can the homeowner

recover from the breaching builder?

Only $4,000, which is the difference between what it cost the homeowner to complete the home

($104,000, including the $100,000 initially paid to the breaching builder and the $4,000 paid to builder X)

and the $100,000 contract price. That $4,000 represents the homeowner’s out of pocket cost to remedy

the initial builder’s deficient performance. The builder repaying the homeowner $4,000 will put the

homeowner in as good a position as performance by the builder would have. That is, a house built for a

cost of $100,000 to the homeowner.

Applying the general formula to this rule, the homeowner’s damages are the value of the promised

performance, $100,000, minus the value of what was received ($100,000 – remember, that the

homeowner in the end received the house he contracted for) (at this point under the formula we are at $0),

plus $4,000 incidental damages paid to builder X as a result of the initial builder’s breach.

Under the alternate, modern approach, the amount needed to make the homeowner whole would still be

$4,000, consisting of a $4,000 reliance interest. As a result of the breached contract, the homeowner

suffered an unanticipated $4,000 cost.

2(b). If instead of paying the builder $100,000 upfront (as described above), the homeowner had initially

paid the builder $60,000, the homeowner would owe the builder the remaining $40,000 less the $4,000

cost of completion (i.e. $40,000 minus the $4,000 cost of completion = $36,000). The court would reason

that the builder “substantially performed” his obligations (thus making the breach “immaterial”),

requiring the homeowner to fulfill her obligation, less the cost of completion.

PROBLEM #3: Repudiation Prior to Performance

Repudiation by Painter on Discounted Performance

A painter contracted with a homeowner to paint the homeowner’s house. The value of such a paint job

was $1,200, but the painter needed work and agreed to do the job for $900. Before the homeowner paid

any money to the painter, the painter then repudiated the contract.

3(a). If the homeowner is then forced to hire another painter to complete the job for $1,100 (a price

reflecting a $100 discount for what we’ve been told was the value of performance), the homeowner

cannot recover $900 from the repudiating painter, but only the loss incurred as a result of entering a

substitute contract.

That is, the homeowner contracted to receive a painted house for $900. As a result of the breach, she

received the painted house for $1,100. Since she is entitled to be placed in as good a position as if the

first painter performed (a painted house for $900), the court will award her damages of $200 to reflect the

difference between what she paid as a result of the breach ($1,100) and the original contract price ($900).

Page 12: CONTRACTS MNEMONICS - Pieper Bar Review › ... › CWS15-Contracts-Mnemonics.pdfcontracting parties (i.e., accepted a K offer arising from the 3PB K) A – Commenced a breach of K

©2015 Pieper Bar Review 12

3(b). If after the painter’s repudiation, the homeowner decided not to paint her house, she is still entitled

to damages her “lost bargain.” That is, contract law allows her to recover as if she had decided to hire

another painter. The law sets the price of this substitute transaction at the market price (which the parties

can establish through expert testimony). So in this situation, the homeowner would be entitled to recover

the difference between the market value of the job ($1,200) and what the homeowner would have had to

pay the painter under the contract ($900), resulting in an expectation damages claim of $300 for the

homeowner’s lost bargain. Another way of looking at the same problem would be to award the

homeowner the value of the performance ($1,200), less the value of what was received ($0), less the cost

of paying the painter $900 (a cost avoided), for a total of $300.

3(c). If the painter painted the homeowner’s house defectively (the fact pattern would have to tell you the

performance was worth, for example, only $700), then the homeowner’s damages in the event she did not

hire someone to fix the paint job, are similar to the scenario above, $500 (the difference between the value

of the properly performed paint job ($1,200) and the value of what the homeowner received (a $700 paint

job)).

PROBLEM #4: Construction Contract Where Homeowner Breaches

No Recovery for costs that could have been avoided

A builder agreed to build a house for $100,000. When the builder had completed $80,000 of the project,

the homeowner repudiated the contract and said she would not pay the builder. The builder then

completed construction of the home, incurring an additional $5,000 cost. What can the builder recover

from the homeowner?

Consider that the contract price was $100,000, the builder’s reliance interest was $85,000, and his

expectation interest was $15,000, but we don’t award damages for “CAPS” (the mnemonic above).

A) $100,000

B) $ 95,000

C) $ 80,000

D) $ 85,000

It cost the builder $85,000 to construct a $100,000 house. From these figures, a court can determine that

the builder would have made a profit of $15,000 (his expectancy interest). However, the builder cannot

recover the $5,000 expended after the homeowner repudiated the contract, because under contract law a

party who stubbornly continues to perform after the other party has repudiated cannot recover losses that

could have been avoided ($5,000).

Thus, the answer is B, $95,000, which is the value of what was promised ($100,000), less what was

received ($0), plus any consequential or incidental damages ($0), less the $5,000 cost that could have

been avoided by stopping work after the repudiation.

Under the alternate, modern approach, the amount needed to make the homeowner whole would still be

$95,000, consisting of an $80,000 reliance interest for costs incurred prior to the repudiation, plus the

builder’s $15,000 lost profit expectancy interest. As with the explanation above, the builder cannot

recover costs that could have been avoided.

Page 13: CONTRACTS MNEMONICS - Pieper Bar Review › ... › CWS15-Contracts-Mnemonics.pdfcontracting parties (i.e., accepted a K offer arising from the 3PB K) A – Commenced a breach of K

©2015 Pieper Bar Review 13

PROBLEM #5: Emotional Distress Arising Out of a Breached Contract

A builder agreed to construct a house for a homeowner. The builder knew when the contract was made

that the homeowner was in delicate health and the new house was of great importance to her. When the

house was done, the homeowner inspected it while the builder waited outside. When the homeowner

came out, she slammed the front door and the whole house collapsed. Can the homeowner recover for her

emotional distress because of the builder’s breach of contract?

No. Such damages ordinarily are not allowed in a breach of contract action.

Keep in mind that there are rare exceptions to this rule, where the court determines that a severe

emotional disturbance was not only foreseeable, but a likely result of a breach. The limited circumstances

in which a court has awarded damages for a severe emotional disturbance arising out of a breached

contract include breached contracts for the burial of a family member, where a messenger is aware of the

contents of a death notification and fails to make a timely delivery resulting in a relative missing a

funeral, where a person agrees to be filmed for television on condition that her face is blacked out but his

face is shown, etc. John Edward Murray, Jr., Murray on Contracts, § 124 (5th ed. 2011).

PROBLEM #6: Waste

Damages for Immaterial Breach = Cost of Completion (Unless Waste)

When there has been a substantial performance (“HAIL”) in good faith, but a defect exists (especially one

which is only incidental to the main purpose of the K), the a court will usually award the cost of

completion to the non–breaching party (see Problem #2 above). However, if the cost to correct this minor

deficiency is drastically large in proportion to the overall contract price, such that completion to the exact

terms of the contract would constitute “economic waste,” the court will award an alternate measure of

damages to protect the immaterially breaching party. This alternate measure of damages is the difference

between the value of the property as constructed and the value of the property if performance had been

properly completed.

For example: A homeowner hired a contractor to build a home for $100,000 and the specifications

required, among other things, that the pipes be “galvanized, lap welded pipe of the grade known as

'standard pipe' of Reading manufacture.” The contractor completed the house perfectly, other than that he

mistakenly used nearly identical pipe manufactured, not by the Reading Manufacturing Company, but by

the Cohoes Rolling Mill Company. The contractor and homeowner agreed that the contractor had

substantially performed the contract and that the breach was immaterial, but the homeowner sought

damages in the amount of $40,000 (the cost to complete the contract, which would require ripping–up and

then refinishing large portions of the house). Here, the courts would say that replacing the Cohoes pipe

with identical Reading pipe (the only difference being a stamp on the exterior of the pipe) would

constitute economic waste. Therefore, the courts would award as an alternate measure of damages: the

difference in value between the house with Cohoes pipe that was received, and the same house with

Reading pipe that was called for in the contract. Jacobs & Young v. Kent, 230 N.Y. 239 (1921); City

School Dist. v. McLane Constr. Co., 85 A.D.2d 749 (3d Dep’t 1981); Essay #1, Issue 2, February 2012;

Essay #1 July 1998 Exam (damages – cost of completion).

However, courts will not award this alternate measure of damages just because the cost of completion is

high. If the defect was not incidental to the contract, and the breaching party did not finish the contract,

the cost of completion will be awarded even though the difference in value damages might be lower. For

example, the owner of a 26–acre industrial site entered a contract with a demolition contractor to sell the

scrap metal from its buildings and equipment for $275,000. The contract also required the demolition

Page 14: CONTRACTS MNEMONICS - Pieper Bar Review › ... › CWS15-Contracts-Mnemonics.pdfcontracting parties (i.e., accepted a K offer arising from the 3PB K) A – Commenced a breach of K

©2015 Pieper Bar Review 14

contractor to re–grade the property to make the property more suitable for resale. The demolition

contractor removed the buildings and equipment, but failed to re–grade the property, and the owner sued

for breach of contract. The owner sought the ordinary measure of damages, i.e. the cost of completion

(the cost to re–grade the property), which was $110,000. The demolition contractor argued that this was

waste, offering proof that the plaintiff–owner could sell the un–graded property for only $3,000 less than

if the property was re–graded (difference in value damages). The court ruled that the demolition

contractor owed the $110,000 cost of completion for the breach, noting “[Defendant–contractor’s]

completed performance would not have involved undoing what in good faith was done improperly, but

only doing what was promised and left undone.” American Standard, Inc. v. Schectman, 80 A.D.2d 318

(4th Dep’t 1981).

PROBLEM #7: Consequential Damages Must Be Foreseeable

A carpenter entered a $10,000 contract to renovate a homeowner’s bathroom by July 1. Based on this

contract, the carpenter entered a separate contract to buy a car for $10,000, to be delivered on July 2. The

car contract provided that if payment for the car was not made on July 2, the cost of the car would

increase to $12,000. The carpenter renovated the homeowner’s bathroom but was not timely paid.

Consequently, the carpenter could not pay for the car on July 2, and was required to pay the additional

$2,000 for the car at a later date.

The carpenter cannot recover the additional $2,000 from the homeowner, because the breaching party is

only liable for those consequential losses that were foreseeable based on the available information at the

time of contracting.

Page 15: CONTRACTS MNEMONICS - Pieper Bar Review › ... › CWS15-Contracts-Mnemonics.pdfcontracting parties (i.e., accepted a K offer arising from the 3PB K) A – Commenced a breach of K

©2015 Pieper Bar Review 15

CONTRACT OFFERS

Effective Acceptance Terminated Revocation

Unilateral

Contract

Offer

When communicated

to the offeree.

If the offer cannot be

accepted by a promise

(exception UCC Sale of

Goods contracts), it is

accepted when contract

performance is fully

completed. Under

Restatement (Second)

Contracts section 45,

acceptance occurs when

an offeree tenders or

begins performance, but

performance must be

completed as a condition

to any recovery.

Passage of stated

time or reasonable

time, by a

counteroffer, an

offeree’s rejection

of the offer,

incompetency of

either party, or by

an intervening

illegally.

Common Law and

New York permit

revocation anytime

before performance is

fully completed.

Restatement (Second)

Contracts § 45 makes

the offer irrevocable

once performance has

been tendered or the

offeree begins

performance.

Bilateral

Contract

Offer

When communicated

to the offeree.

When acceptance of the

offer is dispatched

(“mailbox rule”), or

acceptance is orally

communicated to the

offeror.

Passage of stated

time or reasonable

time, by a

counteroffer, an

offeree’s rejection

of the offer,

incompetency of

either party, or by

an intervening

illegally.

Communicated to the

offeree before

acceptance has been

dispatched.

Options

(Irrevokable

Offers)

When communicated

to the offeree.

Options generally

must be supported by

consideration

(exception GOL

section 5–1109 &

UCC section 2–205 where a signed

writing takes the

place of

consideration).

Acceptance of the option

(called exercising the

option) is effective only

when the acceptance is

received by the offeror.

Passage of the

stated date or after

a reasonable time,

destruction of the

subject matter or

intervening

illegality.

Not revocable by

offeror, and not

revoked by offeror’s

death, or offeree’s

rejection or

counteroffer.

It can be revoked by

the offeror if

revocation notice

reaches offeree before

the option does.

Auction

Offers

(UCC 2–328)

The bid price (offer)

is announced by the

bidder to the

auctioneer.

Auction hammer falls and

auctioneer announces

“SOLD.”

A higher bid

terminated a

pending bid.

The bid is orally

withdrawn by the

bidder before the

hammer falls.

Rejection of

the Offer

Only when it is

received by the

offeror.

Implied when offeror

receives rejection.

Acceptance of offer

is communicated to

the offeror before

the offeror receives

the rejection.

Offeree’s acceptance

is mailed, received or

is communicated to

the offeror before the

rejection is received

by offeror.

Page 16: CONTRACTS MNEMONICS - Pieper Bar Review › ... › CWS15-Contracts-Mnemonics.pdfcontracting parties (i.e., accepted a K offer arising from the 3PB K) A – Commenced a breach of K

©2015 Pieper Bar Review 16

Pre–Existing Duty Rule Summary

Common Law Contracts: A price change had to be supported by new

consideration, otherwise the promise to pay the increased price was

unenforceable for lack of consideration. The parties’ original price would

govern.

UCC Sales Contracts §2-209(1): The agreed price increase is

enforceable without any new consideration if based on “good faith” reason

for asking for more money to perform the contract. No signed writing is

necessary unless the modified total price is $500 or more, which then

requires a signed writing.

Restatement (Second) Contracts §89: The agreed price increase is

enforceable without any new consideration if based on a good faith reason

that was not anticipated when the parties originally entered the contract.

No signed writing is required to enforce the modification.

New York General Obligations Law §5-1103: In contracts that do not

involve the sale of goods, the General Obligations Law abolishes the

Preexisting Duty Rule, provided the price modification is contained in a

writing signed by the party to be charged with breach of the modified

contract.

Page 17: CONTRACTS MNEMONICS - Pieper Bar Review › ... › CWS15-Contracts-Mnemonics.pdfcontracting parties (i.e., accepted a K offer arising from the 3PB K) A – Commenced a breach of K

©2015 Pieper Bar Review 17

In-Class Multistate Questions:

Contracts

Formation, Performance, Breach, and

Discharge (50%) What Concepts Do They Expect You to Know?

I. Formation of Contracts

A. Mutual assent

1. Offer and acceptance

2. Indefiniteness or absence of terms

3. Implied–in–fact contract

4. “Pre–contract” obligations based on

reliance

B. Consideration

1. Bargain and exchange and substitutes

for bargain: “moral obligation,”

reliance, and statutory substitutes

2. Modification of contracts: preexisting

duties

3. Compromise and settlement of claims

II. Performance, Breach, and Discharge

A. Conditions

1. Express

2. Constructive

3. Obligations of good faith and fair dealing

in performance and enforcement of

contracts

4. Suspension or excuse of conditions by

waiver, election, or estoppel

5. Prospective inability to perform: effect

on other party

B. Impracticability and frustration of purpose

C. Discharge of contractual duties

D. Express and implied warranties in sale–of–

goods contracts

E. Substantial and partial breach and

anticipatory repudiation

50 % Defenses to

enforceability, Parol Evidence and

Interpretation, Remedies, and

Third-party Rights

50% Formation of

Contracts, Performance, Breach, and Discharge

Breakdown of Contracts on the MBE

Page 18: CONTRACTS MNEMONICS - Pieper Bar Review › ... › CWS15-Contracts-Mnemonics.pdfcontracting parties (i.e., accepted a K offer arising from the 3PB K) A – Commenced a breach of K

© 2015 Pieper Bar Review 18

Defenses, Parol Evidence and Interpretation,

Remedies, and Third-party Rights (50%) What Concepts Do They Expect You to Know?

I. Defenses to Enforceability

A. Incapacity to contract

B. Duress

C. Undue influence

D. Mistake, misunderstanding

E. Fraud, misrepresentation, and nondisclosure

F. Illegality, unconscionability, and public policy

G. Statute of frauds

II. Parol Evidence and Interpretation

III. Remedies

A. Measure of damages for breach; protecting the expectation interest

B. Consequential damages: causation, certainty, and foresee ability

C. Liquidated damages and penalties

D. Avoidable consequences and mitigation of damages

E. Rescission and reformation

F. Specific performance; injunction against breach; declaratory judgment

G. Restitutionary and reliance recoveries

H. Remedial rights of breaching parties

IV. Third-party Rights

A. Third–party beneficiaries

1. Intended beneficiaries

2. Incidental beneficiaries

3. Impairment or extinguishment of third–

party rights

4. Enforcement by the promisee

B. Assignment of rights and delegation of duties

Page 19: CONTRACTS MNEMONICS - Pieper Bar Review › ... › CWS15-Contracts-Mnemonics.pdfcontracting parties (i.e., accepted a K offer arising from the 3PB K) A – Commenced a breach of K

© 2015 Pieper Bar Review 19

1. A homeowner advertised his home for

sale at an asking price of $100,000. A

prospective buyer expressed an interest in

buying it at the asking price if the

homeowner would paint the exterior.

The homeowner agreed, but told the

buyer that his attorney was out of town

for two weeks and couldn’t draw up the

papers immediately. The parties agreed

that the homeowner would contract to

have the house painted in the meantime.

The next day, the buyer met the painting

contractor and selected an unusual shade

of purple paint for the house. Ten days

later, when the painting contractor was

nearly finished painting the house, the

buyer received a termination notice from

his employer. The buyer immediately

informed the homeowner that he would

no longer be able to buy the house.

The homeowner approached several

realtors regarding the listing of his home,

but the realtors informed him that he

would need to have the exterior

repainted, since the color the buyer had

chosen was highly unattractive to most

buyers.

If the homeowner sues the buyer to

recover his costs in painting the house, is

the homeowner likely to prevail?

(A) No, since the homeowner was not

bound under an enforceable contract

to have the house painted.

(B) No, since the painting did not

constitute an unjust enrichment to

the buyer.

(C) Yes, since the buyer breached the

contract for the sale of the house.

(D) Yes, under a quasi–contract theory,

since the homeowner relied to his

detriment on the buyer’s promise to

purchase the house.

1. Although the painting of the house was

part of the contract for sale which should

have been in writing, courts allow

recovery on a quasi–contract theory when

one party to an oral agreement has

performed or partly performed in reliance

on the contract. Here, the lead in to the

question tells us that the homeowner

incurred “costs,” and the reason he

incurred those costs was the oral

agreement with the buyer. Thus, (A) is

an incorrect answer.

While the homeowner cannot compel

specific performance of the entire

agreement, he can recover the amount he

actually expended. Thus, the answer is

(D), not (C).

The basis of recovery is not unjust

enrichment, so (B) is incorrect.

Page 20: CONTRACTS MNEMONICS - Pieper Bar Review › ... › CWS15-Contracts-Mnemonics.pdfcontracting parties (i.e., accepted a K offer arising from the 3PB K) A – Commenced a breach of K

© 2015 Pieper Bar Review 20

2. Which of the following scenarios would

warrant a quasi–contractual recovery?

(A) A fierce blizzard surprised a cross

country skier as he skied through

the forest. To avoid freezing to

death, the skier broke into a nearby

chalet, burned the owner’s

firewood, and left after the storm

subsided. The owner sued the

skier for the fair rental value of the

chalet and for the fair market value

of the firewood.

(B) A plane crash rendered a passenger

unconscious for 24 hours. A

physician at the scene of the crash

rendered medical assistance to the

unconscious passenger and later

sued the passenger for the value of

his medical services.

(C) A homeowner hired a landscaper to

maintain his lawn at 12 Oak Park

Drive. The landscaper parked his

truck, which has an elaborate sign

identifying his lawn service

company, in front of the

homeowner’s neighbor’s house at

14 Oak Park Drive, and mistakenly

mowed the neighbor’s lawn while

the neighbor watched from her

window. The landscaper sued the

neighbor for the fair value of the

lawn mowing services.

(D) An elderly uncle told his niece that

he would give his house to her if

she would live with him and take

care of him for one year. At the

conclusion of the year, the uncle

refused to give his house to the

niece, and the niece, unable to

obtain specific performance

because of the statute of frauds,

sued her uncle for the fair market

value of the property.

2. A quasi–contract, as distinguished from

an implied–in–fact contract, occurs

when there is no basis for concluding

that the parties by their behavior

impliedly entered into a contract. In

(A) and (C), the skier and neighbor

impliedly agreed by their conduct to

pay for the services rendered to him.

Thus, each is liable under the theory of

an implied–in–fact contract, rather than

quasi–contract.

In (B), the passenger has not by his

conduct impliedly agreed to pay for the

services rendered. Since he was

unconscious, he could not have

knowingly agreed to or acquiesced to

the services provided by the physician.

Therefore (B) is the correct answer.

The situation described in (D) also

gives rise to quasi–contract (i.e, where

an express contract fails because of the

statute of frauds, incapacity of one of

the parties, illegality, mutual mistake,

etc., then in order to avoid the unjust

enrichment of one of the parties, a

quasi–contract may be imposed). The

measure of damages in (D), however, is

incorrect; it is not the value of the

property, but rather the fair market

value of the niece’s services which will

be awarded by the court.

Page 21: CONTRACTS MNEMONICS - Pieper Bar Review › ... › CWS15-Contracts-Mnemonics.pdfcontracting parties (i.e., accepted a K offer arising from the 3PB K) A – Commenced a breach of K

© 2015 Pieper Bar Review 21

3. A student had just completed his junior year

of college and was looking for a summer job.

He was approached by a homeowner, who

said to him, “I have twenty gallons of paint in

my garage. If you will paint my house with it

during the next three weeks, I will pay you

$1,000.” The student immediately went to

the hardware store and purchased three paint

brushes for $20. That night, the homeowner

telephoned the student and said, “I’m sorry,

but my nephew arrived yesterday, and he

needs the work badly, so I won’t need you to

paint my house.”

If the student sues the homeowner for breach

of contract, is the student likely to prevail?

(A) Yes, because once an offeree

commences work pursuant to a

unilateral contract offer, the offer

becomes irrevocable.

(B) Yes, because the homeowner induced

the student to purchase the paint

brushes, and promissory estoppel would

effectively convert the homeowner’s

offer into an option contract.

(C) No, because an offeror may revoke a

unilateral contract at any time before it

is accepted by completion of the act

requested.

(D) No, because the homeowner revoked his

offer before it was accepted.

3. An offer for a unilateral contract becomes

irrevocable when the offeree tenders

performance or commences the work

requested, but not when there is only mere

preparation to perform that work. The

purchase of paint brushes is only preparation

and therefore the homeowner had the right to

revoke the offer at the time he did. (D) is the

correct answer.

(C) is an overly broad statement of the

homeowner’s right to revoke. He could not

revoke at any time prior to completion, but

only before work on the requested project

commenced. Choice (C) would be the

correct choice under New York law.

Peterson v. Pattberg, 248 NY 86 (1928). The

performing party has a cause of action for

restitution in quasi contract, but no claim for

breach of contract.

(B) is incorrect because the reliance in this

case, the purchase of $20 worth of paint

brushes, was not of such substantial character

that the doctrine of promissory estoppel

under §87(2) of the Restatement (Second) of

Contracts would convert the offer into an

option contract. The courts will only award a

remedy on the grounds of promissory

estoppel where the reliance was substantial

(e.g., a substantial expense or forgoing

substantial alternatives) and foreseeable.

Here the economic injury suffered is not of

unconscionable proportion and enforcement

of the contract would not be “necessary to

avoid injustice.” This answer choice may be

tempting since it provides an equitable

solution to the student, but it is incorrect (the

bar examiners like to provide answer choices

that suggest a fair outcome on equitable

grounds to throw you off).

(A) is incorrect because the student had not

begun the work.

Here, homeowner revoked the offer before

the student either tendered performance or

commenced the paint job. The student had

merely prepared to start the paint job.

Calamari, Calamari and Perillo on Contracts

§ 2.22 at 93–94 (6th ed. 2009).

Page 22: CONTRACTS MNEMONICS - Pieper Bar Review › ... › CWS15-Contracts-Mnemonics.pdfcontracting parties (i.e., accepted a K offer arising from the 3PB K) A – Commenced a breach of K

© 2015 Pieper Bar Review 22

4. A store owner received a written offer

from a clothing wholesaler to purchase

600 Oxford style, white, cotton shirts

for $2 per shirt, totaling $1,200. The

wholesale price for similar shirts is

typically $20 per shirt. The store owner

thought that the wholesaler must have

lost his mind, but, nevertheless, agreed

to the offer. The parties each signed a

purchase order which called for

delivery of the shirts to the owner’s

place of business and payment of the

purchase price on July 1. Two days

later, the wholesaler called the owner

and told him that he had made a

mistake because he had read from the

wrong line of his price list. He said that

unless the owner paid him $20 per shirt,

the price of the shirt on the correct line

of the price list, or a total price of

$12,000, he would not deliver the

shirts. The owner refused to pay

anything more than $1,200. When the

wholesaler failed to deliver the shirts on

July 1, the owner purchased 300

comparable shirts for $20 per shirt and

brought suit against the wholesaler for

damages resulting from the

wholesaler’s breach of contract.

Which of the following will the court

conclude?

(A) The wholesaler will prevail

because he made a unilateral

mistake.

(B) The wholesaler will prevail

because of the doctrine of mutual

mistake.

(C) The wholesaler will prevail since it

was obvious to the owner on May

1 that the wholesaler made a

blatant mistake in quoting the price

of $2 per shirt.

(D) The owner will prevail because he

accepted the offer.

4. A contract can be rescinded for

unilateral mistake such as this one only

when the unilateral mistake was so

obvious that the other party must have

known that the first party made a

mistake.

This question clearly indicates that a

unilateral mistake was made by the

wholesaler, but that fact alone is not

sufficient reason to permit him to

rescind the contract. Because (C) is

more precise, (A) is not the best

answer.

(B) is clearly incorrect because there

was no mutual mistake by both parties.

There is only a unilateral mistake.

(C) is the correct answer. There is a

substantial disparity between the

contract price of $2 per shirt and the

market value of the shirts at wholesale,

which seems to be $20 a shirt. If the

buyer, because of the disparity in price,

was aware or should have been aware

that the seller had made a mistake, the

seller will be able to rescind the

contract and, therefore, will prevail.

(D) is incorrect because the owner will

not prevail since he was aware of the

mistake.

Page 23: CONTRACTS MNEMONICS - Pieper Bar Review › ... › CWS15-Contracts-Mnemonics.pdfcontracting parties (i.e., accepted a K offer arising from the 3PB K) A – Commenced a breach of K

© 2015 Pieper Bar Review 23

5. On March 18, a department store published

an ad in the local newspaper that stated:

“Now that winter is behind us, we will be

selling our inventory of forty mink coats

(which have a retail price of $2,000 per

coat) for $600 each, at 8:30 a.m.,

Saturday, March 20. One coat per

person. First Come First Served.”

The owner of a nearby fur store decided to

purchase one of the coats to hold in her

inventory until next fall. She was the first

customer in line when the department store

opened its doors on March 20. She

tendered $600 in cash and asked to

purchase one of the advertised mink coats.

The manager of the fur department

recognized her as the owner of a competing

fur store and refused to sell her a coat.

The owner of the fur store brought an

action against the department store seeking

a judgment that it deliver a coat to her in

exchange for $600, or, in the alternative,

for damages resulting from the department

store’s failure to sell her a coat.

In her action against the department store,

will the owner of the fur store prevail?

(A) Yes, on either remedy, because her

tender of the $600 was a valid

acceptance of the department store’s

offer.

(B) Yes, because the department store

breached a valid contract, but the

owner of the fur store will only be

able to collect money damages.

(C) No, on both remedies, because an

advertisement to the general public is

only an invitation to make an offer,

and when the owner of the fur store

made the offer, the department

store properly refused to accept it.

(D) No, on both remedies, because by

placing the “one coat per person”

restriction in the ad, the department

store was reserving its right to

refuse to sell coats to competitors.

5. An advertisement indicating a present

intention to sell to an individual who

meets specific criteria (for example,

being one of the first customers at the

store on the day of the sale) constitutes

an offer. The owner of the fur store

properly accepted this offer by arriving

at the store and tendering $600 for the

coat. She will not obtain specific

performance, however, because the coat

was not unique. There were forty in

stock at the store, and damages will be

a sufficient remedy. She will be

entitled to recover the difference

between the fair market value of the

coat and $600. Therefore, (B) is correct

and (A) is incorrect.

(C) is incorrect. Although most

advertisements do not constitute offers,

the criteria of this particular

advertisement were specific enough to

constitute an offer.

The advertisement was not to the

general public but only to the first forty

customers who accepted. Customers

reasonably understand that they have

the power to accept when an

advertisement identifies the quantity to

be sold. Murray on Contracts §35 at

79 (5th ed. 2011).

(D) is incorrect because the “one coat

per person” limitation does not

reasonably indicate to a purchaser that

there is any limitation on who may

purchase a coat.

Page 24: CONTRACTS MNEMONICS - Pieper Bar Review › ... › CWS15-Contracts-Mnemonics.pdfcontracting parties (i.e., accepted a K offer arising from the 3PB K) A – Commenced a breach of K

© 2015 Pieper Bar Review 24

6. A series of seven arsons occurred at a city’s

homeless shelter. On January 15, the city

council adopted a resolution that stated:

The city will pay $15,000 for the arrest and

conviction of anyone found guilty of the

seven arsons at the city’s homeless shelter.

The city council proceedings were telecast

live over the city’s local access cable

channel. No other publicity was

disseminated by the city.

On January 25, the city council passed a

resolution repealing its reward offer because

the city was facing budget constraints. The

January 25 proceedings were not telecast

over the local access cable channel because

the channel was televising the city’s high

school basketball tournament that night. The

city council documented the resolution in the

minutes of its proceedings, and the minutes

were published in the legal notices in the

back pages of the local paper.

On February 1, a bar patron overheard a

conversation implicating a disturbed

electrician in the arsons. Since he knew of

the reward, but had not heard of its

revocation, the patron relayed the information

to the police. The police lawfully arrested the

electrician who confessed and was

subsequently convicted of the arsons. The

bar patron sought to recover the reward

money, but the city refused.

If the bar patron sues the city to recover the

reward, will he prevail?

(A) Yes, because he knew of the January 15

resolution at the time he talked to the

police and had no knowledge of the

January 25 resolution repealing the

city’s offer.

(B) Yes, because he had no knowledge of

the January 25 resolution repealing the

city’s offer and his knowledge of the

January 15 resolution is irrelevant.

(C) No, because the January 25 resolution

effectively revoked the offer.

(D) No, because the offer was not accepted within a reasonable time.

6. The January 15 resolution was a unilateral

contract offer which could be accepted by

anyone with knowledge of that offer. It was

a specific offer for a specific crime as

contrasted with a standing offer to reward for

any particular crime, e.g., “$10,000 to anyone

who provides information leading to the

capture of any killer of a police officer.”

This latter type offer can be claimed even if

the offeree was unaware of it.

Restatement (Second) Contracts § 23

provides that “[s]tanding offers of rewards

made by governmental bodies [are] intended

to create a climate in which people do certain

acts in the hope of earning unknown

rewards.” Illustration 3 under § 23 explains,

“A city ordinance provides a standing reward

of [$50,000] will be paid for information

leading to the arrest and conviction of anyone

guilty of [any] arson within the city limits. A

furnishes such information. A is entitled to

the reward whether or not he knew of the

reward or was motivated by hope of the

reward.” Id. Here, the January 25 resolution

revoking the earlier offer was not publicized

in such a way as to revoke the offer except as

to those who had actual knowledge of the

revocation. (A), which incorporates both of

these concepts, is the correct answer.

(B) is incorrect because it suggests that the

bar patron could have claimed the reward

even if he had not known of the January 15

reward offer. A person who acts without

knowledge of an offer is generally incapable

of enforcing a contract. The only exception

is standing offers of reward made by the

government.

(C) is incorrect because the January 25

council proceeding was not televised. Since

the purported revocation did not receive

publicity comparable to the January 15 offer,

it would be ineffective. Murray on Contracts

§ 43 at 121 (5th ed. 2011); Calamari, Calamari and Perillo on Contracts § 220(d)

at 81–82 (6th ed. 2009).

(D) is incorrect because the offer was

accepted within two weeks from the time it was made, which is a reasonable time for a

reward offer to remain outstanding.

Page 25: CONTRACTS MNEMONICS - Pieper Bar Review › ... › CWS15-Contracts-Mnemonics.pdfcontracting parties (i.e., accepted a K offer arising from the 3PB K) A – Commenced a breach of K

© 2015 Pieper Bar Review 25

7. Prior to the start of an auction, each

bidder and the auctioneer signed

documents sufficient to satisfy the

statute of frauds should a contract be

formed at the auction, and the

auctioneer announced that the auction

would be “without reserve.” The

auctioneer then began to auction a

valuable Picasso painting. The first

bidder bid $100,000 for the painting. A

second bidder immediately thereafter

bid $125,000. One minute later, the

second bidder withdrew his bid. No

further bids were made and nothing

relevant was said during the next five

minutes. At that time, the auctioneer

announced that the auction sale was

terminated.

Which of the following is a correct

statement of the parties’ legal rights and

duties?

(A) There is a binding contract

between the auctioneer and the

second bidder for $125,000.

(B) There is a binding contract

between the auctioneer and the first

bidder for $100,000, because the

second bidder’s withdrawal

revived the prior bid.

(C) The auctioneer is not contractually

bound to sell the painting, because

no bids were made within a

reasonable time of the second

bidder’s revocation.

(D) The auctioneer is not contractually

bound to sell the painting, because

he could withdraw the painting

from auction at any time before his

hammer fell.

7. Auctions of personal property are

governed by UCC §2–328.

(A) is incorrect because, under UCC

§2–328(3), a bidder may retract his bid

at any time before the auctioneer’s

hammer falls.

(B) is incorrect because, under UCC

§2–328(3), the retraction of a bid does

not revive a prior bid.

If the auction sale is “without reserve,”

the auctioneer cannot withdraw the

article from auction. (D) is therefore

incorrect.

However, if there are no bids within a

reasonable time, an auctioneer can

terminate an auction that is “without

reserve.” Since this occurred, (C) is

correct.

Page 26: CONTRACTS MNEMONICS - Pieper Bar Review › ... › CWS15-Contracts-Mnemonics.pdfcontracting parties (i.e., accepted a K offer arising from the 3PB K) A – Commenced a breach of K

© 2015 Pieper Bar Review 26

8. On June 1, a property seller mailed a signed

letter to a buyer which stated:

“I will sell you my house at 23 Garden Lane in

Louisville for $150,000. I want a deposit of

$15,000 and will close at the registry of deeds

on August 1st. Please reply by June 15.”

On June 3, the buyer wrote back:

“Thank you. Would you consider selling for

$145,000?”

On June 5, the seller wrote to the buyer:

“I have received your letter of June 3. I reject

your offer to buy my house for $145,000.”

On June 7, the buyer wrote to the seller:

“I have received your letter of June 5, and

accept the offer contained in your letter of

June 1. I am enclosing herewith my check in

the amount of $15,000 as the deposit you

requested. Please deliver a deed to the

property conveying marketable title at the

closing on August 1, at which time I will pay

to you the balance of the purchase price.”

On June 9, the seller wrote to the buyer:

“I am returning herewith your check for

$15,000. I am not contractually obligated to

sell you my house at 23 Garden Lane,

Louisville.”

The buyer appeared at the Louisville registry of

deeds on August 1, prepared to tender the entire

$150,000 purchase price, but the seller did not

appear. The buyer brought an action against the

seller for specific performance of the contract.

Will the buyer likely prevail?:

(A) No, because his June 3 letter was a

counter–offer.

(B) No, because the seller’s June 5 letter

revoked his June 1 offer.

(C) No, because the buyer’s June 7 letter was

not an effective acceptance because it

contained additional terms.

(D) Yes, the buyer will prevail.

8. (A) is incorrect because the June 3 letter

would best be characterized as an inquiry.

An inquiry does not constitute a counter–

offer and therefore is not a rejection of the

original offer.

(B) is incorrect because the June 5 letter was

a reply to the inquiry of June 3, and did not

by its terms revoke the June 1 offer. This

choice incorrectly characterizes the June 3

letter as a counter–offer, when it was really

only an inquiry.

(C) is incorrect because the additional terms

set forth in the June 7 letter were terms that

were implied in the original offer (namely,

that the seller convey marketable title). As

we will cover in further detail in real

property, if a contract for the sale of real

property makes no mention of the quality of

title to be conveyed, it is implied in the

contract (not in the deed) that the seller will

tender marketable title at closing.

(D) therefore is correct because the buyer

accepted the seller’s June 1 offer though his

June 7 letter.

Page 27: CONTRACTS MNEMONICS - Pieper Bar Review › ... › CWS15-Contracts-Mnemonics.pdfcontracting parties (i.e., accepted a K offer arising from the 3PB K) A – Commenced a breach of K

© 2015 Pieper Bar Review 27

9. A university advertised for bids for a new ice

hockey arena. The bids were to be submitted

June 1 and opened on June 15, at which time

the contract would be awarded. One of the

major items included in the bid was a large

compressor.

A general contractor planned to bid on the

project and asked several compressor dealers

to bid on the required compressor. He

indicated to the dealers that if he won the

contract, he would enter into a contract to buy

the compressor from the lowest bidder on

June 20, with delivery to take place on

August 20. On May 15, a large compressor

dealer submitted a written offer to sell to the

contractor the required compressor for

$75,000. This was the lowest offer received

by the contractor. On May 20, the contractor

called the dealer, stated his intent to bid on

the university ice hockey arena, and asked if

the dealer would hold his offer open until

June 21. The dealer said yes. The contractor

then submitted a $5,000,000 bid to construct

the ice hockey arena.

The university awarded the contract to the

contractor, who was the lowest bidder, on

June 15. On June 16, the dealer learned that

the contractor received the contract,

immediately called the contractor, and

revoked the offer to sell the compressor for

$75,000. On June 17, the contractor wrote to

the dealer and accepted the offer to sell the

compressor for $75,000. When the dealer did

not deliver the compressor on August 20, the

contractor purchased the compressor

elsewhere for $90,000. In an action by the

contractor against the dealer to recover the

$15,000 difference between the purchase

price and the amount of the dealer’s offer,

will the contractor prevail?

(A) Yes, because the dealer’s offer was a

“firm” offer under the Uniform

Commercial Code and could not be

revoked until June 21.

(B) Yes, because the dealer’s agreement to

keep the offer open until June 21 was

enforceable since the dealer knew that

the offer would induce substantial reliance on the part of the contractor.

(C) Yes, because a written offer must be

revoked by a writing.

(D) No, because the dealer validly revoked

his offer before the contractor accepted

it.

~ ~ ~ ~ ~

9. A similar scenario was presented in

Essay #1 on the February 2001 New

York Bar Exam.

(B) is correct. The offer made by the

dealer to the contractor is governed by

the rules of promissory estoppel set forth

in §87(2) of the Restatement (Second) of

Contracts. When the dealer made the

offer, he knew it would induce substantial

reliance by the contractor in bidding on

the skating rink contract. Therefore, an

option contract was created and the

option was properly exercised by the

contractor.

(A) is wrong because the promise to keep

an offer open under UCC §2–205 must be

in writing. Here, the dealer’s promise to

keep the offer open was oral.

(C) is wrong because the offer, if it was

revocable at all, could have been revoked

by a telephone call.

(D) is incorrect because the offer was not

revocable before June 21 for the reasons

set forth in (B).

Page 28: CONTRACTS MNEMONICS - Pieper Bar Review › ... › CWS15-Contracts-Mnemonics.pdfcontracting parties (i.e., accepted a K offer arising from the 3PB K) A – Commenced a breach of K

© 2015 Pieper Bar Review 28

10. On March 16, an artist mailed to a

patron an offer to paint the patron’s

portrait for $2,500. On April 1, the

patron mailed back his acceptance, but

conditioned the acceptance upon the

painter framing the portrait in a frame

worth at least $500.

On April 2, the patron found in his attic

a frame perfect for the portrait. That

same day, the patron faxed to the artist

an acceptance of the artist’s offer and

advised the artist that he had a suitable

frame in which to put the painting.

On April 3, the artist received the

patron’s April 1 correspondence.

What is the status of the parties

relationship on April 4?

(A) There is no contract to paint the

portrait for $2,500 because of the

“mailbox rule.”

(B) There is no contract to paint the

portrait because the patron’s

counter–offer rescinded the artist’s

offer.

(C) There is a contract to paint the

portrait for $2,500, and for the

artist to supply a $500 frame.

(D) There is a contract to paint the

portrait for $2,500 but no

obligation for the artist to supply a

$500 frame.

10. A rejection terminates the offeree’s

power of acceptance. A counter–offer

(a new proposal), because it is an

implied rejection of the offer, has the

same effect as an express rejection.

Restatement (Second) Contracts § 39.

Thus, (C) is not correct because the

artist’s obligation to obtain the frame

was a counter–offer that the artist never

agreed to.

(B) otherwise would be correct except

for the fact that a rejection (the April 1

letter) is effective only when received

(April 3) by the offeror (the artist). (B)

is not correct because, before the artist

received the counter–offer (rejection),

he received the April 2 acceptance by

fax. Thus, there was a binding contract

as of April 2 and (D) is correct.

(A) is incorrect because the mailbox

rule states that an acceptance is

effective when dispatched and put out

of the offeree’s control (dropped into

the mailbox), and it applies to an

“acceptance” and not a rejection of the

offer (a counter–offer). A rejection is

not effective when dropped into the

mailbox, but rather only when received

by the offeror, here the artist.

Page 29: CONTRACTS MNEMONICS - Pieper Bar Review › ... › CWS15-Contracts-Mnemonics.pdfcontracting parties (i.e., accepted a K offer arising from the 3PB K) A – Commenced a breach of K

© 2015 Pieper Bar Review 29

11. The owner of a chain of fast food

restaurants entered into a written contract

with a manager to operate one of her

restaurants for two years at a salary of

$52,000 per year, payable at the rate of

$1,000 per week. After the manager had

worked at the restaurant for 16 months,

he suggested to the owner a way of

recycling the fat used to cook french fries

which would result in significant savings

in both purchase and disposal costs for

the entire restaurant chain. The

suggestion was implemented immediately

and was very successful.

Thereafter, the owner told him that she

was so pleased with his fat recycling

suggestion, that she was raising his pay to

$2,500 per week for the remainder of the

contract.

When the manager’s next paycheck

arrived, it was only for $1,000. He

approached the owner and she told him

that, after talking with her accountant,

she had decided not to raise his pay, but

would continue to honor the conditions of

the original contract.

If the manager brings an action against

the owner to secure the additional $1,500

per week, will he prevail?

(A) Yes, because he conferred a material

benefit on the owner.

(B) Yes, because his promise to manage

the restaurant for the remainder of

the contract term would make the

owner’s promise enforceable.

(C) No, because past consideration

would not support the owner’s

promise.

(D) No, because a contract for more than

a year in duration must be in writing

to be enforceable.

11. The suggestion to recycle fat was not part

of any bargain between the manager and

the owner, and neither past nor “moral”

consideration is sufficient to support the

owner’s promise to pay the manager

more money for the job he was already

contractually obligated to perform for

$1,000 per week. Since this contract is

governed by common law rules, new or

additional consideration is necessary to

support a modification. Therefore, (C) is

correct. See Pieper NYAA p. 247 (2014).

(A) is incorrect because the conferring of

a material benefit, if not bargained for,

does not support a contract. The benefit

conferred by the manager was not

“bargained for” as part of the owner’s

promise since it already had been

performed by the manager.

(B) is incorrect because the manager is

already contractually bound to manage

the restaurant for the remainder of the

term, and his promise to live up to his

existing contract cannot furnish

consideration for a modification of that

contract.

(D) is incorrect because, at the time the

owner promised to increase the

manager’s salary, the contract had less

than one year to run and therefore the

promise would not be unenforceable

because of the statute of frauds.

Page 30: CONTRACTS MNEMONICS - Pieper Bar Review › ... › CWS15-Contracts-Mnemonics.pdfcontracting parties (i.e., accepted a K offer arising from the 3PB K) A – Commenced a breach of K

© 2015 Pieper Bar Review 30

12. A talented sailor was tied for the lead in a

prestigious weekly race series, with one week

remaining before the final race of the season.

Unfortunately, his boat began to leak and

required emergency repairs if the sailor

expected to compete in the final race. A local

carpenter told the sailor that because his

regular boss did not have any work for him

for the coming week, he would repair the

boat for $1,200 paid in advance. The sailor

paid the $1,200 on the condition that the

repairs be completed so that the boat could

sail the following weekend, and the carpenter

began the repairs.

On Thursday morning the carpenter informed

the sailor that his regular boss needed him for

an emergency job on Thursday and Friday.

His boss had offered to pay him $500 per day

to work for him, so the carpenter told the

sailor he would not finish the boat by

Saturday unless the sailor paid him an

additional $300 on Saturday and agreed not

to sue the carpenter for his anticipatory

breach of contract. Because the sailor was

anxious to win the race series, he agreed to

the carpenter’s proposal. The carpenter

finished the boat by Saturday morning.

When the sailor took possession of the boat,

he refused to pay the carpenter the additional

$300 promised on Thursday.

If the carpenter brings an action against the

sailor in a common law jurisdiction, will he

be able to recover the $300?

(A) Yes, because the emergency excused the

carpenter’s original performance

deadline, and his promise to complete

the job despite the emergency was valid

consideration for the second agreement.

(B) Yes, because the sailor’s promise not to

sue for the anticipatory breach of

contract was a bargained–for exchange

for the carpenter’s promise to complete

the job on time.

(C) No, because of the preexisting duty rule.

(D) No, because the carpenter’s conduct was

unconscionable.

12. The carpenter was under a legal duty to

complete the repairs by Saturday. Since the

facts state that the action was brought in a

common law jurisdiction (and not in a

jurisdiction recognizing the Restatement

(Second) of Contracts) and the contract was

not a contract for the sale of goods governed

by the Uniform Commercial Code,

consideration is required for a modification

of the contract. The carpenter is doing

exactly what he originally promised to do,

and, therefore, there is no consideration to

support the promise to pay the additional

$300. Therefore, (C) is correct.

(D) is an attractive choice but wrong, because

the conduct of the carpenter is not sufficiently

unconscionable. He originally agreed to do

the work at a low price because he was out of

work, and he started the job in good faith. It

was only because his regular boss called him

and offered him more money that he asked

for additional compensation for the sailor’s

job. The preexisting duty rule clearly applies

here, making (C) the better answer.

(A) is incorrect because of the preexisting

duty rule. The carpenter’s promise is to do

exactly what he is already legally obligated to

do.

(B) is incorrect because the sailor is not

bargaining with the carpenter to give up his

lawsuit in exchange for agreeing to pay the

carpenter an additional $300. Both of these

promises are to the sailor’s detriment, and

there is no new promise to the carpenter’s

detriment

Page 31: CONTRACTS MNEMONICS - Pieper Bar Review › ... › CWS15-Contracts-Mnemonics.pdfcontracting parties (i.e., accepted a K offer arising from the 3PB K) A – Commenced a breach of K

© 2015 Pieper Bar Review 31

13. A man applied for and obtained a

$100,000 life insurance policy. The

policy contained the following

language:

“This policy shall not insure against a

death which occurs when the insured

is scuba diving, unless he is diving

with a certified P.A.D.I. instructor.”

Two months after the policy was

issued, the man was scuba diving in the

company of the senior dive instructor.

The instructor was certified by S.S.I., a

training organization similar to

P.A.D.I., and had 30 years of scuba

diving experience. While the man and

the instructor were diving, a great white

shark appeared and attacked the man,

killing him instantly. Prior to this

attack, no great white sharks had been

spotted in or near those waters for more

than one hundred years.

The beneficiary of the policy bought an

action to collect on the policy, but the

insurance company refused to pay.

What is the insurance company’s best

defense?

(A) The death would not have occurred

if the insured had the benefit of a

certified P.A.D.I. instructor’s

experience during the fatal dive.

(B) Scuba diving is a hazardous

activity properly excluded from the

policy.

(C) The insured failed to comply with

a contractual condition.

(D) The purpose of the policy

exclusion was to protect the

company against the type of scuba

diving fatality which occurred.

13. (C) is the correct answer. The language

quoted from the policy is a condition

precedent to performance, which must

be satisfied before there can be a

recovery under the contract. American

Home Assur. Co. v. International Ins.

Co., 90 N.Y.2d 433, 442–443 (1997);

Security Mut. Ins. Co. v. Acker–

Fitzsimons Corp., 31 N.Y.2d 436

(1972).

(B) is incorrect because scuba diving is

not excluded from the policy. Only

diving without a certified P.A.D.I.

instructor will result in application of

the exclusion.

(A) and (D) are also incorrect. Whether

the statements are true or whether the

death would have resulted under

different conditions is irrelevant. The

specific language of the condition is

controlling.

Thus, a condition precedent to the

performance of this contract (payment

of money) was that any drowning death

by scuba diving be in the presence of

P.A.D.I. instructor. Perillo, Calamari

& Perillo on Contracts §11.5 at 362

(6th ed. 2009).

Page 32: CONTRACTS MNEMONICS - Pieper Bar Review › ... › CWS15-Contracts-Mnemonics.pdfcontracting parties (i.e., accepted a K offer arising from the 3PB K) A – Commenced a breach of K

© 2015 Pieper Bar Review 32

14. A homeowner and a painter entered into

a contract to paint the homeowner’s

house for $700 prior to a wedding that

was being held in the homeowner’s

backyard. The contract specifically

stated that payment was contingent

upon completion by the wedding date.

The next day, the painter was offered a

more lucrative job, so the painter told a

handyman that he was the owner of the

house, and that he needed the house

painted by the wedding day for $500.

The painter paid the handyman, who

began work on the job, but failed to

complete the job on time.

If the homeowner brings an action

against the painter and the handyman,

from whom can he recover damages?

(A) The painter only.

(B) The handyman only.

(C) Both the painter and the

handyman.

(D) Either the painter or the handyman,

at the homeowner’s election, but

not both.

14. The homeowner has a valid cause of

action against the painter because the

painter was a party to the original

contract and was not discharged from

his obligations when he delegated the

work.

On the other hand, the handyman, who

is really a subcontractor, is not liable to

the homeowner because the two are not

in privity. Indeed, the painter told the

handyman that the painter was the

owner of the house, so the handyman

had no reason to know of the actual

homeowner’s interest in the completion

of the paint job. Moreover, because the

painter did not relay the urgency of

painting the house, it is unlikely that the

painter can recover from the handyman

as time was not made “of the essence”

in the contract between the painter and

the handyman.

(A) is therefore correct.

Page 33: CONTRACTS MNEMONICS - Pieper Bar Review › ... › CWS15-Contracts-Mnemonics.pdfcontracting parties (i.e., accepted a K offer arising from the 3PB K) A – Commenced a breach of K

© 2015 Pieper Bar Review 33

15. A patient owed her physician $25,000 for

professional services. The physician orally

assigned this claim to her adult daughter as

a wedding gift. Shortly thereafter, after

suffering sudden, severe losses in the stock

market, the doctor assigned the same claim

to her stockbroker in a signed writing, in

partial satisfaction of advances legally

made by the stockbroker. Subsequently,

the patient, without knowledge of either

assignment, paid the physician the $25,000

then due, which the physician promptly lost

at a horse track, although she remains

solvent.

Assuming that Article 9 of the Uniform

Commercial Code does NOT apply to

either of the assignments in this situation,

which of the following is a correct

statement of the parties’ rights and

liabilities?

(A) As the assignee prior in time, the

physician’s daughter can recover

$25,000 from the patient, who acted at

her peril in paying the physician.

(B) As the sole assignee for value, the

stockbroker can recover $25,000 from

the patient, who acted at her peril in

paying the physician.

(C) Neither the physician’s daughter nor

the stockbroker can recover from the

patient, but the physician’s daughter,

though not the stockbroker, can

recover $25,000 from the physician.

(D) Neither the daughter nor the

stockbroker can recover from the

patient, but the stockbroker, though

not the physician’s daughter, can

recover $25,000 from the physician.

15. Generally, an assignment terminates the

assignor’s right to collect the contract

benefit (here, the account receivable), but

the assignee, absent a filing of a UCC

Article 9 financing statement, must

immediately notify the obligor owing the

money (here, the patient) of the

assignment. Since neither assignee notified

the patient, she satisfied her debt by paying

the physician. Choice (D) is the correct

answer.

Choice (A) is not the correct answer

because, even though the “first–in–time”

rule generally prevails, it does not apply

when the first assignee was gratuitous. A

gratuitous assignment is revocable until the

gratuitous assignee gets paid, recovers a

judgment against the person owing the

money, or enters a new agreement with the

obligor. Thus, the physician’s daughter has

no claim against either the physician or the

patient since her rights were revoked by (1)

the reassignment of the same right, as well

as (2) the payment of the debt to the

physician. Choices (A) and (C) are

incorrect.

Although past consideration makes the

assignment to the stockbroker non–

gratuitous, because the stockbroker did not

notify the patient, the patient’s payment of

the debt to the physician (the assignor)

extinguished the patient’s obligation. Thus,

choice (B) is incorrect.

Choice (D) is the best choice. Even though

the physician’s daughter’s gratuitous

assignment right was extinguished by the

physician’s reassignment of the same right

to the stockbroker, and the stockbroker’s

right against the patient was extinguished

by his failure to give notice to the patient

prior to her making payment in full to the

physician, none of this extinguished the

original $25,000 claim the stockbroker had

against the physician which was supported

by consideration. Thus, the stockbroker

has a claim against the physician.

Page 34: CONTRACTS MNEMONICS - Pieper Bar Review › ... › CWS15-Contracts-Mnemonics.pdfcontracting parties (i.e., accepted a K offer arising from the 3PB K) A – Commenced a breach of K

© 2015 Pieper Bar Review 34

16. A farmer entered into a contract to sell

her entire tobacco crop to a buyer for

$100,000, to be paid by the buyer upon

delivery. Thereafter, the farmer

purchased a race horse from a breeder,

and, in writing, assigned her right to

payment from the buyer to the breeder

as payment for the horse. The race

horse was diseased when the breeder

delivered it to the farmer and it died the

next day.

If delivery of the diseased race horse

constituted a breach of warranty of

merchantability by the breeder, what is

the farmer’s right vis–a–vis the

assigned rights to the tobacco crop?

(A) The assignment to the breeder from

the farmer would be revoked

automatically.

(B) The farmer could revoke the

assignment to the breeder by

giving him notice.

(C) The farmer could only revoke the

assignment through an action for

rescission.

(D) The farmer could not revoke the

assignment by any procedure.

16. Choice (C) is the correct answer. An

assignment for consideration is in effect

a transfer of title to, or ownership of, a

contract right. The transferee, or

assignee, takes title to this contract right

to collect money. In order to revoke the

assignment, an action for rescission

would have to be commenced. The

farmer does have a right to rescind here,

because she had a right to reject the

race horse, however, it must be

accomplished through court action.

The farmer cannot revoke the

assignment by any other method.

Therefore, choices (A) and (B) are

incorrect.

Choice (D) is incorrect because the

farmer may revoke the assignment.

Page 35: CONTRACTS MNEMONICS - Pieper Bar Review › ... › CWS15-Contracts-Mnemonics.pdfcontracting parties (i.e., accepted a K offer arising from the 3PB K) A – Commenced a breach of K

© 2015 Pieper Bar Review 35

17. A real estate investor entered into an

agreement for a builder to construct a

six bedroom summer home and a three

bedroom summer home that the

investor wanted to rent out for the

summer months. The investor was to

pay for each house when each was

completed. The builder agreed to have

the six bedroom home completed by

March 15, and the three bedroom home

completed by March 30. A pipe burst

in the three bedroom home during

construction and destroyed all of the

rooms but the den. The six bedroom

home was not completed until June 15,

which was too late to take advantage of

the summer rental market.

The real estate investor refused to pay

the builder for the six bedroom house

when the bill was tendered on June 25.

Will the builder succeed in its breach of

contract action against the real estate

investor?

(A) No, because the law implies that

the time set forth in the contract is

of the essence.

(B) Yes, because the contract did not

specifically provide that time was

of the essence.

(C) Yes, because the flood in the

three–bedroom home would extend

the time for completion of the six–

bedroom home.

(D) Yes, under the common law, but

not if the Uniform Commercial

Code was enacted by the

jurisdiction.

17. In contracts other than UCC contracts,

time usually is not of the essence unless

the parties make it an express condition

of the contract or the circumstances

indicate that the parties intended time to

be of the essence.

(A) is not correct. “A party [generally]

need not perform on the precise day

stated in the contract unless time is

made of the essence.” Calamari,

Contracts § 11.18 at 376 (6th ed. 2000).

Note, however, although usually

construction contracts are not made

time of the essence, they may be of the

essence even though it is not expressly

stated. It depends on the parties’ intent,

and the circumstances. Id. at n.24.

Since the circumstances here did not

indicate that time was of the essence,

and the contract did not specifically

provide that time was of the essence,

(B) is the best answer.

Since the two contracts were divisible,

the flood in the three bedroom house

would extend its completion date, but it

would not affect the completion date for

the six bedroom house. Thus, (C) is an

incorrect answer.

The UCC does not apply to building

contracts. Thus, (D) is an incorrect

answer.

Page 36: CONTRACTS MNEMONICS - Pieper Bar Review › ... › CWS15-Contracts-Mnemonics.pdfcontracting parties (i.e., accepted a K offer arising from the 3PB K) A – Commenced a breach of K

© 2015 Pieper Bar Review 36

18. The owner of a female horse that won

horse racing’s most prestigious event

was unable to breed the horse when its

racing career ended. The owner entered

into a written contract to sell the horse

for $50,000 to a veterinarian, who

truthfully described himself as an avid

horse racing fan. $50,000 was a

generous price for a sterile

thoroughbred, but far less than what the

owner could have obtained if the horse

had been capable of reproducing.

Using his expertise in breeding animals

that had previously been incapable of

conceiving, the veterinarian extracted

eggs from the horse and fertilized them

in vitro. When the owner learned that

the horse was pregnant, he brought an

action to rescind the sale.

Will the owner prevail?

(A) No, because he has no basis to

rescind this valid contract.

(B) No, because of mutual mistake.

(C) Yes, because the veterinarian failed

to disclose that he specialized in

the breeding of previously infertile

animals.

(D) Yes, because his ignorance of the

fact that the horse was capable of

being bred successfully constitutes

unilateral mistake.

18. In this case, the veterinarian saw an

opportunity to use his unique skills to

create great value from a retired race

horse. He was not mistaken concerning

the ability of the horse to produce

offspring. Therefore, there was no

mutual mistake and (B) is incorrect.

While the veterinarian probably knew

that the owner thought that the horse

would never have offspring, it was not a

proven fact at the time of the sale that it

could not reproduce. The facts do not

indicate that it was pregnant at the time

of the sale. A mistaken belief about

possible future events does not form the

basis for rescission for unilateral

mistake. This doctrine is reserved for

an obvious mistake of fact known at the

time the contract was completed. (D) is

therefore incorrect.

The owner and the veterinarian

bargained at arms length. Although the

owner did not disclose his plans, he

made no misrepresentation. He is not,

in fact, obligated to make full

disclosure of his plans. Therefore, the

owner may not rescind the contract for

fraud and (C) is incorrect.

(A) is correct because there is no basis

to rescind this contract. See the two

cases in the NYAA pp. 594-595 (2014).

Page 37: CONTRACTS MNEMONICS - Pieper Bar Review › ... › CWS15-Contracts-Mnemonics.pdfcontracting parties (i.e., accepted a K offer arising from the 3PB K) A – Commenced a breach of K

© 2015 Pieper Bar Review 37

19. An elderly man whose health was

declining relied on his nephew to

manage his finances. The nephew

would fill out checks for the elderly

man and then the elderly man would

sign them.

The nephew suggested that the elderly

man purchase a boat, and after many

discussions, convinced the elderly man

to purchase the nephew’s boat for

$20,000, an amount substantially more

than the boat was worth.

Does the elderly man have a valid basis

to vacate the transaction?

(A) Yes, based on misrepresentation.

(B) Yes, based on duress.

(C) Yes, based on undue influence.

(D) Yes, based on mistake.

19. Choice (A) is incorrect because the

facts do not indicate that the nephew

induced the elderly man to enter the

transaction based on any

misrepresentation or that the elderly

man relied on (was deceived by) any

misrepresentation.

Mistake as to value generally is not a

basis to rescind a contract. Thus, the

best answer is not choice (D).

The elderly man cannot successfully

argue duress because there were no

economic or other threats brought to

bear on him. Recognize that the theory

of duress only permits a party to avoid a

transaction where a wrongful act or

threat overcomes the free will of that

party and results in an oppressive or

abusive contract. Here, the elderly man

could have easily elected not to

purchase the boat. Thus, choice (B) is

not the best choice.

Undue influence speaks of “unfair”

persuasion by one party misusing a

position of trust and confidence, or by

using a dominant psychological

position in an unfair manner.

Unfairness may be established by the

excessive or inadequate consideration

involved. Choice (C) is the best choice.

Page 38: CONTRACTS MNEMONICS - Pieper Bar Review › ... › CWS15-Contracts-Mnemonics.pdfcontracting parties (i.e., accepted a K offer arising from the 3PB K) A – Commenced a breach of K

© 2015 Pieper Bar Review 38

20. A builder agreed to construct two

cottages for an owner. One cottage was

to have three bedrooms and the price

agreed upon was $80,000. The other

was to have two bedrooms and the price

was $75,000. Upon completing each

cottage, the owner was to pay the

builder in full. The owner provided the

plans and specifications, and told the

builder that he was going to rent the

houses for the summer months. The

builder agreed to complete the two–

bedroom cottage by March 1, and the

three–bedroom cottage by March 15.

The builder began construction on the

three bedroom cottage on January 5,

and construction on the two–bedroom

cottage on February 1. On February 20,

after the three–bedroom cottage was

three–fourths completed, it was

destroyed by fire through the fault of

neither the builder nor the owner. The

builder completed the two–bedroom

cottage on February 26.

If the builder refuses to rebuild the

three–bedroom cottage, will he be able

to recover on the contract for the

completed two–bedroom cottage?

(A) Yes, because the builder did not

have an obligation to rebuild the

three–bedroom cottage.

(B) Yes, only if the owner waives his

claim for the breach of the contract

for the three–bedroom cottage.

(C) Yes, because the contract is

divisible.

(D) Yes, because of the doctrine of

impossibility of performance.

20. Since the builder was to receive

separate compensation for building

each of the two houses, this was a

divisible contract. Even if a party’s

performance falls short of that required

by the doctrine of substantial

performance, a court can avoid

forfeiture and allow recovery on the

contract by holding that the contract is

divisible (or severable) rather than

entire. Under the doctrine of divisible

contract, a party who is in breach can

nevertheless recover for part

performance.

Even if the builder does not build the

$80,000 cottage, he has substantially

performed his obligation to build the

$75,000 cottage and so should be

allowed to recover for that cottage

under the contract. Thus, (C) is the

correct answer.

Because the builder can recover for the

$75,000 cottage regardless of his

performance of the other part of the

contract, (A), (B), and (D) are incorrect

answers.

Note, the owner has a claim for

damages against the builder for the

builder’s failure to complete the three–

bedroom cottage. Restatement

(Second) of Contracts §240, illus. 1.

Page 39: CONTRACTS MNEMONICS - Pieper Bar Review › ... › CWS15-Contracts-Mnemonics.pdfcontracting parties (i.e., accepted a K offer arising from the 3PB K) A – Commenced a breach of K

© 2015 Pieper Bar Review 39

21. A contractor entered into a written contract

to construct a house for a homeowner in

accordance with certain plans and

specifications for $95,000, payable in

installments as the work progressed. No

lien law was applicable in the jurisdiction.

As the house neared completion and the

contractor had been paid $90,000, the

contractor informed the homeowner that his

credit was shaky with plumbing supply

company which sold him plumbing

supplies. He explained that he would be

unable to obtain delivery of $3,000 worth

of sinks and toilets to complete the project,

unless he paid the company $8,000 which

he owed for the plumbing supplies

previously purchased for the homeowner’s

house.

The homeowner thereafter called the

company on the telephone and told its

manager that if he delivered the $3,000

worth of sinks and toilets which the

contractor had ordered for the house, the

homeowner would act as a surety for the

contractor’s outstanding obligation to pay

the $8,000.

The manager agreed and shipped the sinks

and toilets. Thereafter, the contractor filed

for bankruptcy without having made any

payment to the plumbing supply and

without completing the house for the

homeowner.

If the plumbing supply company brings an

action against the homeowner, will the

plumbing supply company prevail?

(A) No, because the homeowner’s promise

to pay the $8,000 is unenforceable

because of the statute of frauds.

(B) No, because the contractor was

already obligated to pay for the

plumbing supplies, and therefore the

homeowner’s promise was not

supported by consideration.

(C) Yes, and it will recover $3,000.

(D) Yes, and it will recover $8,000.

21. A promise to pay the debt of another

ordinarily must be in writing to be

enforceable due to the statute of frauds.

However, there is an exception under the

“main purpose rule.” If the main purpose of

the guarantee is to further the goals of the

guarantor, the statute of frauds is

inapplicable. That rule is applicable here,

because the main purpose of the

homeowner’s promise was to get his house

finished. Restatement (Second) of

Contracts §116 illus. 3 (1981). Therefore,

(A) is incorrect.

(B) is incorrect because the manager’s

shipment of the goods was valid

consideration to support the homeowner’s

promise to pay the debt.

The promise made by the homeowner was

to pay the entire $8,000 due for plumbing

materials used in the construction of his

house. Once the main purpose rule takes

the promise out of the statute of frauds, that

promise is enforceable. Therefore, the

homeowner owes the entire $8,000, not just

the $3,000 for the fixtures shipped, and

(D), not (C), is the correct answer.

Page 40: CONTRACTS MNEMONICS - Pieper Bar Review › ... › CWS15-Contracts-Mnemonics.pdfcontracting parties (i.e., accepted a K offer arising from the 3PB K) A – Commenced a breach of K

© 2015 Pieper Bar Review 40

22. When is the court most likely to invoke

the parol evidence rule?

(A) When it discovers evidence of fraud.

(B) When parties seek to introduce

evidence concerning a consistent

term in a fully integrated contract

with a merger clause.

(C) When parties seek to introduce

evidence concerning a consistent

additional term on a partially

integrated contract.

(D) When parties seek to introduce

evidence explaining the trade

meaning of a critical contract term in

a fully integrated contract.

22. “A writing that is final is an integration

of the terms embodied in it. When it is

final and complete it is a total

integration. A writing that is final, but

does not completely express the parties’

contract is a partial integration.” Perillo,

Calamari & Perillo on Contracts §3.2 at

107 (6th ed. 2009).

“Thus, a partial integration may not be

contradicted by what has been called

‘parol evidence.’ A total integration not

only cannot be contradicted by the type

of evidence in question but cannot even

be supplemental by consistent (non–

contradictory) additional terms.” Id.;

Restatement (Second) Contracts §210(1)

and Comment a.

“A merger clause states that the writing is

a final, complete, and exclusive statement

of all of the terms agreed on. Williston’s

first rule, which is followed by most

courts, is that a merger clause will

ordinarily resolve the issue of total

integration.” Perillo, Calamari & Perillo

§3.6 at 122 (6th ed. 2009). Thus choice

(B) is the best choice for invoking the

parol evidence rule.

The general rule is that parol evidence is

admissible to show fraud in the

inducement. Thus, choice (A) is not

correct.

A partial integration, being final but

incomplete, may be supplemented by

consistent additional terms.” Perillo,

Calamari & Perillo § 3.2 at 107 (6th ed.

2009). Thus, the parol evidence rule

would not be a bar to choice (C).

UCC 2–202 provides that even a writing

intended by the parties as a final

expression of their agreement may be

explained or supplemented by a course of

dealing, by trade usage or by a course of

performance. Thus parol evidence could

be used in choice (D).

Page 41: CONTRACTS MNEMONICS - Pieper Bar Review › ... › CWS15-Contracts-Mnemonics.pdfcontracting parties (i.e., accepted a K offer arising from the 3PB K) A – Commenced a breach of K

© 2015 Pieper Bar Review 41

23. A restaurant owner and a contractor

entered into an agreement for

construction of a restaurant. The plans

and specifications required the

contractor to install an “Airflow

Heating System.” About a week before

the heating system was to be installed,

the contractor discovered that it would

take four months to obtain an “Airflow

Heating System,” but that a “Flowheat

Heating System” was immediately

available. Since, the contractor was

unable to contact the owner to receive

his permission for the change, he

consulted a heating engineer, who

determined that the “Flowheat” system

was as good as the “Airflow” system, it

would heat just as well, and that it was

of somewhat better quality. Relying

upon this advice, the contractor

installed the Flowheat system. The

restaurant owner objected to the

substitution of the Flowheat system and

refused to pay the contractor.

In an action by the contractor against

the restaurant owner for damages,

which of the following would be the

amount of damages recoverable by the

contractor?

(A) The contractor can recover because

of the doctrine of impossibility of

performance.

(B) The contractor can recover under

the doctrine of substantial

performance.

(C) The contractor cannot recover

because he failed to contact the

owner.

(D) The contractor can recover in

quasi–contract.

23. (B) is correct because, under the

doctrine of substantial performance, the

contractor can recover on the contract.

Although the contractor did not

completely or exactly conform to the

specifications of the contract, he

substantially performed. Under the

common law rule, substantial

performance is enough to allow a party

to recover on a contract, though he may

still be liable for damages resulting

from his technical breach. See Essay 3,

July 2008.

To require the contractor to remove the

“Flowheat” system and the cost of

reinstalling an “Airflow” system would

result in economic waste.

(A) is not applicable to this kind of

situation. Generally, the doctrine of

impossibility only applies to situations

where the contract has become illegal,

the subject matter of the contract has

been destroyed, or a party to a personal

services contract has died or become

disabled.

(C) is incorrect because the owner was

not available, so the contractor could

not contact him.

(D) is incorrect because the doctrine of

substantial performance allows

recovery on the contract itself, not in

quasi–contract.

Page 42: CONTRACTS MNEMONICS - Pieper Bar Review › ... › CWS15-Contracts-Mnemonics.pdfcontracting parties (i.e., accepted a K offer arising from the 3PB K) A – Commenced a breach of K

© 2015 Pieper Bar Review 42

24. A renowned financial planner entered

into a written contract with a woman to

prepare an estate plan for a fee of

$10,000, to be paid upon the completion

of the plan. The woman retained the

financial planner because of his

reputation, skill and experience in

financial planning. However, the woman

and the financial planner got along poorly

and, after the second conference, when

only a draft of a plan dealing with life

insurance and real estate had been

submitted to the woman, they had a

profound disagreement about the proper

way to handle the woman’s stock

portfolio. Hoping to avoid such

encounters in the future, the financial

planner, without the woman’s

knowledge, assigned all of his rights and

duties under his financial planning

contract with the woman to an associate

who had recently graduated from college.

The associate expressly promised the

financial planner to carry out the work to

the best of his ability.

The woman, upon learning of the

assignment, refused to allow the associate

to proceed with the financial plan and

brought an action against the financial

planner to compel him to resume and

complete performance of the contract.

Is the woman entitled to such relief?

(A) No, because the financial planner

has contracted to render personal

services to the woman.

(B) No, because the financial planner

effectively delegated his remaining

duties under the contract to the

associate.

(C) Yes, because the financial planner is

a preeminent financial planner and

his services are unique.

(D) Yes, because the financial planner

has personally completed a

substantial portion of the project.

24. The contract for an estate plan is a

personal service, and so a specific

performance remedy is not available. (A)

is therefore correct.

(B) is incorrect because the financial

planner cannot effectively delegate his

obligations under a contract requiring his

skill and expertise.

(C) is incorrect. Although it is the reason

why the delegation to the associate was

improper, it does not support an award of

specific performance.

(D) is incorrect because the financial

planner’s completion of a portion of the

work does not become the basis of a

specific performance remedy for the

remainder.

Page 43: CONTRACTS MNEMONICS - Pieper Bar Review › ... › CWS15-Contracts-Mnemonics.pdfcontracting parties (i.e., accepted a K offer arising from the 3PB K) A – Commenced a breach of K

© 2015 Pieper Bar Review 43

25. A seller agreed to sell 5,000 widgets to

a buyer for $15,000. The goods were to

be delivered on or before December 15.

On December 5th, while the goods

were in transit but before the goods

were delivered, the buyer called the

seller and told him that she did not want

the goods and would not accept them

because she had purchased them

cheaper somewhere else.

What right did the buyer’s telephone

call give the seller?

(A) The right to stop the goods in

transit if they were sent in carload

lots.

(B) The right to stop the goods in

transit even if they were not sent in

carload lots.

(C) The right to stop delivery of the

goods even if they had reached the

buyer’s city and the carrier had

acknowledged to the buyer that it

was holding them for her.

(D) The right to stop the goods in

transit only if the buyer is

insolvent.

25. (A) is the correct answer because the

buyer’s phone call constituted an

anticipatory breach of the contract

between the buyer and seller. Since this

is a contract for the sale of goods, UCC

Article 2 applies. Under Article 2, a

seller has the right to stop goods in

transit when there has been an

anticipatory breach, but only if they

were sent in carload lots (for example,

an entire freight car of widgets).

Therefore, (B) is incorrect.

(D) is incorrect because a seller has the

right to stop the goods in transit upon

the buyer’s anticipatory breach

regardless of whether the buyer was

insolvent. Furthermore, the facts do not

indicate that the buyer was insolvent,

but they do indicate that the buyer

anticipatorily breached the contract. If

the buyer was insolvent, the seller

would have the right to stop the goods

in transit whether or not they were sent

in carload lots.

(C) is incorrect because a seller loses

her right to stop goods in transit once a

carrier has acknowledged the buyer’s

right to the goods.

Page 44: CONTRACTS MNEMONICS - Pieper Bar Review › ... › CWS15-Contracts-Mnemonics.pdfcontracting parties (i.e., accepted a K offer arising from the 3PB K) A – Commenced a breach of K

© 2015 Pieper Bar Review 44

26. A buyer and seller entered into a

contract for the purchase and sale of

1,000 widgets at a price of $5 per

widget. The widgets were to be

shipped F.O.B. the buyer’s plant, and

were to be delivered to the plant on or

before December 1. The seller shipped

them out on November 27. Later in the

day on November 27, before the buyer

received the widgets, the buyer called

the seller and informed the seller that,

since widgets were no longer popular

gift items, he was cancelling the order.

The goods were destroyed by fire after

the buyer’s telephone call.

At the time of the fire, which party bore

the risk of loss of the widgets?

(A) The buyer, because the risk of loss

shifted as soon as the seller

delivered the goods to the carrier.

(B) The buyer, because she repudiated

the contract.

(C) The buyer, but only to the extent

that the seller’s insurance failed to

cover the loss.

(D) The seller, because the contract

specified “F.O.B. buyer’s plant.”

26. Ordinarily, the risk of loss passes

entirely to the buyer when the seller

completes his delivery obligations.

Since the contract specifies F.O.B.

buyer’s plant, the risk ordinarily would

not pass until the goods actually reach

the buyer. However, if the buyer

repudiates or breaches the contract, the

risk of loss immediately shifts to the

buyer as to goods that are identified, to

the extent that the seller’s insurance

fails to cover the loss. UCC §2–510(1).

Therefore, (C) is the correct answer,

since the buyer repudiated the contract.

Note, that this same F.O.B. risk of loss

rule was tested on Essay #1 of the July

2003 New York Bar Exam.

(A) is incorrect. Since the contract

specifies F.O.B. buyer’s plant, the risk

of loss did not shift when the seller

delivered the goods to the carrier.

(B) is incorrect because the risk of loss

only passed to the buyer to the extent

that the seller’s insurance did not cover

the loss.

(D) is incorrect because the risk of loss

shifted to the buyer, in part, when he

repudiated the contract.

Page 45: CONTRACTS MNEMONICS - Pieper Bar Review › ... › CWS15-Contracts-Mnemonics.pdfcontracting parties (i.e., accepted a K offer arising from the 3PB K) A – Commenced a breach of K

© 2015 Pieper Bar Review 45

27. A manufacturer of off–road construction

vehicles, entered into a written contract

with a dealer to sell to the dealer three

large trucks for $30,000 each. The

manufacturer was to deliver one truck on

March 1, one on April 1, and one on May

1. Payment for each of the trucks, in the

amount of $30,000, was to be made

within 5 days after each delivery. The

manufacturer delivered the first truck on

March 1.

On March 4, the dealer called the

manufacturer and informed him that,

because he had encountered a delay in

obtaining a bank loan, he could not pay

for the truck delivered on March 1 until

March 20. The dealer, although he had

been a reputable heavy equipment dealer

for 25 years, had not paid for vehicles in

a timely fashion since February 20. The

dealer still has possession of the truck

delivered on March 1.

Which of the following remedies is NOT

available to the manufacturer on March

5?

(A) Immediately demanding a return of

the truck delivered on March 1, and

reclaiming that truck.

(B) Refusing to deliver the truck

scheduled for April 1 delivery unless

the dealer pays for the first truck

before then and pays for the April 1

truck in cash on or before the time of

delivery.

(C) Suing the dealer for the $30,000.

(D) Terminating the contract for the

April 1 and May 1 trucks.

27. The dealer was insolvent between

February 20 and March 20 under the

definition of insolvency under § 1–201 of

the UCC because he could not pay his

debts when they were due. Therefore, the

manufacturer, after demanding the return

of the truck within 10 days of delivery,

could reclaim the truck delivered on

March 1st under § 2–702 of the UCC.

Therefore, (A) is an available remedy and

an incorrect answer.

(B) is also an available remedy and,

therefore, a wrong answer. If an

insolvent buyer defaults on a payment

obligation under an installment contract,

the seller is not obligated to ship in the

future on credit, but can demand payment

in cash and can also demand that the

insolvent buyer pay for any prior

delivered installments before any further

installments are delivered. UCC § 2–

702(1).

(C) is also an available remedy and,

therefore, a wrong answer. Where there

is a breach of the obligation to pay for

one installment, the seller can sue for that

installment prior to completion of all

deliveries, if the amount of each

installment is clear, as it is in this case

($30,000 per installment).

(D) is the correct answer because the

seller cannot terminate the entire

installment contract when the buyer

failed to pay only one installment on

time, even when the buyer is technically

insolvent. “An installment contract is

one which requires or authorizes the

delivery of goods in separate lots to be

separately accepted....” UCC § 2–612.

The seller may, however, demand

assurances of payment before he makes

future deliveries and can require payment

in cash at the time of delivery.

Page 46: CONTRACTS MNEMONICS - Pieper Bar Review › ... › CWS15-Contracts-Mnemonics.pdfcontracting parties (i.e., accepted a K offer arising from the 3PB K) A – Commenced a breach of K

© 2015 Pieper Bar Review 46

28. A buyer and a seller entered into a

written contract for the sale of 10,000

lawnmowers, for delivery on February

1. On January 10, the seller notified the

buyer that due to a fire in its factory, the

factory would remain closed until

February 1, but the seller could

guarantee delivery no later than

February 15. The buyer rejected the

seller’s proposal, immediately

contacted with another lawnmower

distributor to obtain 10,000

lawnmowers at a slightly higher cost,

and brought an action against the seller

to recover the difference between the

cost of the replacement lawnmowers

and the original contract price.

In the buyer’s action against the seller,

will the buyer prevail?

(A) No, because the seller gave

assurances of its willingness and

ability to perform the contract in

February.

(B) No, because the buyer’s lawsuit is

premature before February 1.

(C) Yes, because buyer’s complaint

alleges an actionable tort by seller.

(D) Yes, because buyer’s complaint

alleges an actionable breach of

contract by seller.

28. The notice given by seller to buyer

amounted to an anticipatory breach of

contract by seller, since it

unequivocally stated that it would not

be able to ship conforming goods in the

quantity stated at the time required.

Such an anticipatory breach gives the

buyer, among other remedies, the right

to cover: i.e, the right to purchase the

goods elsewhere and to sue the seller

for the difference between the price the

buyer had to pay for the goods and the

original contract price. The buyer did

this and now has a right to sue in

contract for damages. Thus, (D) is the

correct answer.

The buyer’s suit is based on breach of

contract. The seller has committed no

tort against the buyer. Therefore, (C) is

incorrect.

(A) is incorrect because of the perfect

tender rule under UCC Article 2. This

rule does not allow for a unilateral

delay in performance to a later,

reasonable time when the seller has

unforeseen difficulties.

(B) is incorrect because the buyer does

not have to wait until the date of

performance when there has been an

anticipatory breach.

Page 47: CONTRACTS MNEMONICS - Pieper Bar Review › ... › CWS15-Contracts-Mnemonics.pdfcontracting parties (i.e., accepted a K offer arising from the 3PB K) A – Commenced a breach of K

© 2015 Pieper Bar Review 47

29. On October 1, a buyer entered into a

written contract with a seller for the

purchase of 1,000 mechanical dogs at $20

per dog, to be specially manufactured by

the seller according to the buyer’s

specifications. In order to obtain

operating funds, the seller, as borrower,

entered into a written agreement on

October 5 with a finance company. In

relevant part, this agreement recited,

“Seller hereby transfers and assigns to

the finance company its October 1

mechanical dog contract with Buyer,

as security for a 50–day loan of

$15,000, the advance and receipt of

which are hereby acknowledged by

Seller ....” No copy of the agreement, or

statement relating to it, was filed in an

office of public record.

By November 16, the seller, without legal

excuse, had delivered no dogs, but the

buyer brought no action against the seller.

If the buyer brings an action against the

finance company on account of seller’s

default, will the buyer prevail?

(A) No, because the October 5

assignment by the seller to the

finance company was only an

assignment for security.

(B) No, because no record of the

October 5 transaction between seller

and finance company was publicly

filed.

(C) Yes, because the buyer was a third–

party intended beneficiary of the

October 5 transaction between seller

and the finance company.

(D) Yes, because the October 5

transaction between the seller and

the finance company affected, with

respect to buyer as creditor, a

novation of debtors.

29. UCC 2–210 states that the assignee of a

sale of goods contract impliedly assumes

the full duties of performance under the

terms of that contract unless it is clear

from the circumstances of the assignment

that the assignee did not intend to assume

the delegation of duties.

The purpose of the assignment of the

contract to the finance company was to

give the finance company a security

interest to insure repayment of the loan.

It was only an assignment of the benefits

of the contract, not a delegation of the

duties. The finance company never

expressly or impliedly agreed to perform

the obligations of the contract and

therefore, was not liable for a breach of

the contract. Therefore, (A) is correct.

(B) is incorrect because the filing of the

contract would only be material if there

was a question of priority among

competing security interests, a problem

not present here. The critical issue with

respect to the finance company’s liability

is whether it accepted the obligations of

the contract, which it did not.

(C) is incorrect because there was no

intent to benefit the buyer in the security

assignment made by the seller and the

finance company, and so the buyer was

not an intended third party beneficiary.

(D) is incorrect because there was no

novation –– no agreement by the buyer,

the finance company, and the seller to

substitute the performance of the finance

company for that of the sellers on the

contract.

Page 48: CONTRACTS MNEMONICS - Pieper Bar Review › ... › CWS15-Contracts-Mnemonics.pdfcontracting parties (i.e., accepted a K offer arising from the 3PB K) A – Commenced a breach of K

© 2015 Pieper Bar Review 48

30. A snowmobile manufacturer contracted

with an engine company to purchase

100 engines for $250 each. The

contract of sale expressly required that

each engine would be able to generate

32 horsepower. The contract also

provided that the engines would be

delivered in lots of 25 and that payment

would be made within 24 hours after

delivery.

The first lot of 25 engines was

delivered on June 1, at which time the

snowmobile company forwarded

payment to the engine company in the

amount of $6,250. However, when

later tested, none of the 25 engines

delivered, or any of the 100

manufactured, could generate more

than 15 horsepower.

Despite the lack of sufficient

horsepower, the snowmobile company

installed five of the engines in

snowmobiles. On June 10, the

snowmobile company rejected all of the

engines that had been delivered, and

advised that it would reject all future

deliveries because of the insufficient

horsepower.

On the above facts, how many engines

did the snowmobile company accept?

(A) 0.

(B) 5.

(C) 25.

(D) 100.

30. (B) is correct. The installation of five

engines constitutes acceptance of those

engines because it is a use which is

inconsistent with the seller’s rights.

The phrase “Despite the lack of

sufficient horsepower” is a crucial clue

showing that the buyer was aware that

the goods were non–conforming yet

elected to use five of them with

knowledge of the non–conformity.

(A) is incorrect. The buyer used five of

the engines with knowledge of a defect

and that constituted acceptance.

(C) is incorrect. Each engine is a

distinct commercial unit separable from

the rest of the lot. A buyer may accept

part of a lot and reject the rest, provided

he does not break up a commercial unit.

Here, the buyer accepted five units and

properly rejected twenty. UCC § 2–

601.

(D) is incorrect. The buyer has the

right to accept part of the goods and

reject the rest; hence, acceptance of five

engines did not constitute acceptance of

the total number specified in the

contract.

Page 49: CONTRACTS MNEMONICS - Pieper Bar Review › ... › CWS15-Contracts-Mnemonics.pdfcontracting parties (i.e., accepted a K offer arising from the 3PB K) A – Commenced a breach of K

© 2015 Pieper Bar Review 49

31. A manufacturer entered a contract to

sell a $20,000 printer to a publisher.

Just prior to the delivery date fixed in

the contract, the manufacturer notified

the publisher that it would be unable to

deliver the machine.

Six months thereafter, the publisher

purchased the same printer for $22,000.

The publisher then brought a claim

against the manufacturer for the

publisher’s lost profits of $1,500 per

month for the six months ($9,000) and

the additional $2,000 the publisher had

to pay to for the printer.

At the trial, the manufacturer

introduced unrefuted credible evidence

that if the publisher had purchased a

replacement printer when the

manufacturer breached the contract, the

publisher could have readily purchased

it for $20,000 and would have lost only

one month of profit.

What is the maximum amount the

printer is entitled to recover from the

manufacturer?

(A) $2,000.

(B) $11,000.

(C) $1,500.

(D) $3,500.

31. The traditional measure of damages for

a total breach of contract by a seller is

the difference between the market price

of the goods at the time of the breach

and the parties’ contract price. Under

these facts, there are no breach of

contract damages since the market price

at the time of the contract’s breach

($20,000) was the same as the $20,000

contract price. Thus, neither (A), (B),

nor (D) is correct.

The UCC also permits a buyer to

“cover”, i.e., make a good faith

purchase or enter a contract to purchase

substitute goods without unreasonable

delay. The buyer may then recover the

difference between the cost of cover

($22,000) and the contract price

($20,000).

Here, however, waiting six months to

cover is an unreasonable delay. The

publisher’s “damages are measured as

of the time the buyer could have

covered.... [D]amages cannot be

enhanced by the buyer’s remaining

idle....” Perillo, Calamari & Perillo on

Contracts §14.20 at 515–16 (6th ed.

2009).

Thus, the publisher can only recover

$1,500 in consequential damages, i.e,

up to the time the publisher could have

covered. The publisher cannot recover

for all six months of lost profit.

Therefore, (B) would not be correct.

(C) is the correct choice. “[A] buyer

who fails to cover may be denied

consequential damages that could have

been avoided by cover.” White &

Summers, Uniform Commercial Code §

7–4 at 304 n.43 (6th ed. 2010).

Page 50: CONTRACTS MNEMONICS - Pieper Bar Review › ... › CWS15-Contracts-Mnemonics.pdfcontracting parties (i.e., accepted a K offer arising from the 3PB K) A – Commenced a breach of K

© 2015 Pieper Bar Review 50

32. A coat manufacturer purchased 100

bolts of first quality wool from a wool

producer. The sales contract provided

that the manufacturer would remit

payment prior to inspection. The wool

was shipped, and the manufacturer paid

the producer. Upon inspection, the

manufacturer discovered that the wool

was No. 2 quality. Thereupon, the

manufacturer tendered back the wool to

the producer and demanded return of

his payment. The producer refused,

contending there is no difference

between No. 1 quality wool and No. 2

quality wool.

Which of the following statements

regarding the contract provision

concerning pre–inspection payment is

correct?

(A) It constituted acceptance of the

goods.

(B) It constituted a waiver of the

buyer’s remedy of private sale in

the case of nonconforming goods.

(C) It did not impair the buyer’s right

to inspect the goods prior to

accepting or rejecting.

(D) It required the buyer to “accept”

the goods without inspecting them,

but did not affect the buyer’s right

to revoke acceptance.

32. The basic rule is that a buyer always

has the right to inspect goods before

accepting them. This is true even if the

buyer has agreed to pay for the goods

prior to inspection. Remember to

separate the buyer’s payment from the

right of inspection.

(A) is incorrect because the mere

agreement to pay before inspection in

no way constitutes an acceptance of the

goods. An acceptance, remember,

occurs only when: (1) the buyer

signifies that he or she is accepting; (2)

the buyer uses the goods in some way

inconsistent with the seller’s rights; or

(3) the buyer fails to give timely notice

of rejection. Therefore, (C) is the

correct answer.

(B) is an absurd answer that is clearly

incorrect. There is simply no

connection between the promise to pay

prior to inspection and the buyer’s

remedy of selling the goods in a private

sale.

(D) is also incorrect, as the clause does

not require acceptance before

inspection; it only requires payment.

Page 51: CONTRACTS MNEMONICS - Pieper Bar Review › ... › CWS15-Contracts-Mnemonics.pdfcontracting parties (i.e., accepted a K offer arising from the 3PB K) A – Commenced a breach of K

© 2015 Pieper Bar Review 51

33. A publisher purchased 100 cases of

white 24 lb. paper from a paper

company. The publisher paid for the

paper prior to receipt and the paper

company sent 20 lb. paper instead of 24

lb. The publisher promptly tendered the

paper back to the company and

demanded return of its payment. The

paper company refused, contending the

papers to be identical.

Assuming that the 20 lb. paper is

nonconforming, which of the following

remedies is available to the publisher?

(A) Specific performance.

(B) Damages, measured by the

difference between the value of the

goods delivered and the value of

conforming goods.

(C) Damages, measured by the price

paid plus the difference between

the contract price and the cost of

buying substitute goods.

(D) None, since the publisher waived

his remedies by agreeing to pay

before inspection.

33. (A) is incorrect because specific

performance is only allowed in sale of

goods cases where the property is

unique or in “other proper

circumstances.” There is nothing

unusual about these goods and nothing

to indicate that damages would not

afford a full remedy.

(B) is incorrect because this is the

measure of damages to be used when

the buyer accepts the goods. Here,

however, the buyer rejected them.

(C) states the correct measure of

damages because the publisher already

paid for the goods and then rejected

them as nonconforming. Thus, the

publisher can recover restitution

damages for the price paid plus the

increased cost of cover.

(D) is incorrect because remedies are

never waived by agreeing to pay for the

goods.

Page 52: CONTRACTS MNEMONICS - Pieper Bar Review › ... › CWS15-Contracts-Mnemonics.pdfcontracting parties (i.e., accepted a K offer arising from the 3PB K) A – Commenced a breach of K

© 2015 Pieper Bar Review 52

34. A shoe wholesaler received a catalogue

from a shoe manufacturer which

described all of the manufacturer’s

shoes as “all leather except

ornamentation” and “of the highest

quality.”

Thereafter, the wholesaler and the

manufacturer entered into an agreement

whereby the manufacturer agreed to sell

10,000 pairs of shoes to the wholesaler.

The parties’ contract conspicuously

stated:

“There are no warranties expressly

or impliedly made by seller in

connection with this sale beyond the

description of the goods contained in

the seller’s contract.”

Did the disclaimer in the contract

negate any of the seller’s warranties?

(A) Yes, all express warranties.

(B) Yes, the implied warranty of

fitness for a particular purpose.

(C) Yes, the implied warranty of

merchantability.

(D) No, the disclaimer was ineffective.

34. Choice (A) is incorrect because the

catalogue clearly contains a description

of the goods (“of the highest quality” is

an express warranty). Where an

express written warranty is given, a

clause in the sales contract or the bill of

sale purportedly disclaiming that

warranty must fail.

(C) is incorrect because the magic

words “merchantability”, “as is,” or

“with all faults” do not appear in the

disclaimer clause, and hence the clause

in the contract could not operate as a

disclaimer of that warranty.

The disclaimer clause, however, will

operate to negate any implied

warranties of fitness, as a disclaimer of

that warranty can be made by the use of

general terminology. Therefore (B) is

correct and (D) is incorrect.